M Dosier M3 Trigonometria

75
Dosier módulo III

description

trigonometria,manual de formacion docente,el salvador

Transcript of M Dosier M3 Trigonometria

Page 1: M Dosier M3 Trigonometria

Dosier módulo I

Dosier módulo IIII

Page 2: M Dosier M3 Trigonometria
Page 3: M Dosier M3 Trigonometria

Módulo III

Page 4: M Dosier M3 Trigonometria

1. Introducción

2. Razones y fórmulas trigonométricas 2.1. Ángulos 2.1.1. Medición de un ángulo

77

2.1.2. Terminología 2.2. Razones trigonométricas para ángulos agudos2.3. El círculo unitario2.4. Razones trigonométricas para ángulos destacados 2.5. Iden dades Trigonométricas

2.5.1. Las iden dades básicas 2.5.2. Iden dades de suma y resta de ángulos 2.5.3. Iden dades de duplicación y linearización

2.6. Tabla resumen de iden dades trigonométricas más importantes2.7. Lecturas y ejercicios complementarios

2.8. Problemas

3. Resolución de triángulos y ecuaciones trigonométricas 21 3.1. Ecuaciones trigonométricas

3.1.1. Ecuaciones de la formaA cos (x) +B sen (x) = C

3.2. Lecturas y ejercicios complementarios

4. Transformaciones en el plano: isometrías, traslaciones y rotaciones 27 4.1. Isometrías: Simetrías 4.1.1. Simetrías Centrales 4.1.2. Propiedades de las simetrías centrales 4.1.3. Simetrías Ortogonales 4.1.4. Propiedades de las simetrías ortogonales 4.1.5. Algunas ac vidades para realizar en el aula 4.2. Traslaciones 4.2.1. Ac vidad 4.2.2. Propiedades de las Traslaciones

4.3. Rotaciones 4.3.1. Rotación en el plano orientado

4.3.2. Ac vidad4.3.3. Propiedades Esenciales

Page 5: M Dosier M3 Trigonometria

4.9. Descomposición de traslaciones y rotaciones como composición de simetrías axiales4.10. Problemas de Traslaciones4.11. Problemas de Simetrías Centrales o Reflexiones Puntuales4.12. Problemas de Rotaciones

5. Transformaciones en el plano: homotecias 545.1. Teoremas Fundamentales5.2. Homotecia de figuras geométricas fundamentales 5.3. Problemas de Homotecias5.4. Composición de Homotecias

4.8. La isometría directa más simple4.7. Ac vidad 34.6. Ac vidad 2 4.5. Ac vidad 14.4. Transformaciones en el plano: composición de simetrías y homotecias

Page 6: M Dosier M3 Trigonometria

En este módulo se estudian los aspectos espaciales del mundo físico que pueden ser tratados con el

lenguaje y los conceptos de la trigonometría y las transformaciones geométricas en el plano. A partir de los

conceptos básicos geométricos y algebraicos se abordan los temas básicos de la trigonometría en el plano,

a saber: el estudio de la relación de semejanza entre figuras geométricas, el cálculo de medidas de ángulos

y distancias, las razones e identidades trigonométricas, la resolución de triángulos y la construcción y

visualización de las funciones trigonométricas y sus propiedades. Se hace énfasis en la visualización de las

figuras y los aspectos geométricos subyacentes y, siguiendo el enfoque del curso de geometría, se continúa

haciendo un uso gradual del método deductivo para justificar y formalizar las propiedades y relaciones

básicas de las identidades y funciones trigonométricas. Y con los movimientos o transformaciones

geométricas en el plano se estudia el efecto geométrico de una traslación, una reflexión, un giro, una

homotecia o una combinación de ellas sobre una figura o configuración geométrica.

Page 7: M Dosier M3 Trigonometria

Razones y fórmulas trigonométricas

Page 8: M Dosier M3 Trigonometria

La trigonometría se define como el estudio de las relaciones entre los lados y los ángulos de un triángulocualquiera. Estas relaciones se resumen en las llamadas razones trigonométricas.

En el desarrollo del presente módulo se asume que el estudiante está familiarizado con el trabajo contriángulos (clasificación, terminología, rectas notables, ángulos dentro de un triángulo, área, perímetro,teorema de Pitágoras) y que ene nociones básicas de trigonometría en triángulos rectángulos.

2.1. Ángulos

Imagínese una línea recta, comenzando de una posición fijaOA (Fig. 1a) y rotando alrededor de un puntoO en la dirección opuesta almovimiento de las agujas del reloj hasta llegar a la posición indicada porOB.Al rotar de OA a OB, se describe el ángulo AOB. De esta manera, tenemos el concepto de un ángulocomo formado por la rotación de un segmento de recta alrededor de un punto fijo, que llamaremos elvér ce del ángulo. Como se puede observar en la figura 1a, si se toma cualquier puntoC del segmento derectaOA, este puntomarcará con sumovimiento el arcoCD de un círculo. No existe límite en la rotaciónde OA y, por consiguiente, se pueden formar ángulos de cualquier magnitud al rotar un segmento derecta de esta manera. Suponga, por ejemplo, que la rotación de OA se con núa hasta la posición OA0

(Fig. 1b), en los queA0,O yA son colineales. El puntoC habrá delineado un semicírculo y el ánguloA0OAse le llama ángulo llano. Si dejamos queOA con nue su rotación hasta que llegue a su posición original,habrá realizado una rotación completa y el punto C habrá marcado la circunferencia de un círculo.

2.1.1. Medición de un ángulo

Por convención, los ángulos se miden en sen do an horario. Cualquier ángulo formado a par r de larotación de un segmento sobre un punto en el sen do de las agujas del reloj tendrá una medición ne-ga va. El concepto de la formación de un ángulo por rotación nos lleva a un método conveniente demedir ángulos. Si imaginamos que la rotación completa se divide en 360 divisiones iguales, tendremos360 pequeños ángulos iguales, cada uno de los cuales será llamado un grado1. Note que ésta medición

1Puede que se esté preguntando el porqué el 360 como el número de divisiones de una rotación completa para obtenerun grado. La selección de este número fue hecha de forma muy temprana en la historia de la civilización humana y sabemos,por ejemplo, a par r de las inscripciones en templos que el número fue empleado por los an guos Babilonios. El número

A

B

OC

D

(a)

AA0O CD

(b) Ángulo Llano

Figura 1

8

Page 9: M Dosier M3 Trigonometria

AA0O CD

(a) Rotación completa

III

III IV

AA0O CD

E

F

(b) Ángulo recto

Figura 2

de ángulos, aunque comúnmente u lizada, no es la única existente. Los gradianes, por ejemplo, dividenal círculo en 400 partes iguales. Esta medición fue adoptada en Francia para que el sistema de medidade ángulos fuese consistente con otros sistemas de medición métricos.

OA

B

� = 1 rad

Figura 3

Un tercer método para medir ángulos es absoluto, en el sen do queno depende de una división arbitraria del círculo formado por una re-volución completa. Para obtener la medición, tómese un círculo concentroO (ver figura 3). Rote el radioOA hasta la posiciónOB, tal quela longitud del arco AB es igual a la del radio. A la medida del ánguloAOB le llamaremos un radián. Note que la descripción anterior nosdice que la medición en radianes de un ángulo está definida como larazón de la longitud del arco definido por la rotación que describe alángulo y la longitud del radio del círculo. Dado que la cirfunferenciase ob ene con 2�r , donde r es el radio del círculo, una revolucióncompleta equivale a un ángulo de 2� radianes.

2.1.2. Terminología

La figura 2b representa una rotación completa, como la mostrada enla figura 2a. SeaE y F los puntosmedios de los arcos entreC yD en cada semicírculo. Si trazamos el seg-mento de recta EF , este pasa a través deO y la circunferencia queda dividida en cuatro partes iguales.Cada uno de los ángulosCOE,EOD,DOF y FOC representa a un cuarto de la rotación completa y sonllamados ángulos rectos, conteniendo cada uno 90 grados. El círculo, además es dividido en cuatro par-tes iguales llamadas cuadrantes que se numeran en el orden de formación. Cualquier ángulo menor que90 grados es llamado agudo y cualquiera que se encuentre entre 90 y 180 grados es llamado obtuso. Dosángulos cuyasmedidas suman 180 y 90 grados son llamados ángulos suplementarios y complementarios,respec vamente. Dos ángulos que enen la misma medida son llamados congruentes.

probablemente surgió de la división de los cielos por astrónomos an guos en 360 partes, correspondientes a lo que se tomabacomo el número de dias en el año.

9

Page 10: M Dosier M3 Trigonometria

2.2. Razones trigonométricas para ángulos agudos

Suponga que � un ángulo agudo, es decir, 0 < � < 90 ó 0 < � <�

2. Con este ángulo agudo, se puede

formar un triángulo rectángulo con hipotenusa de longitud c y catetos de longitudes a y b, tal como semuestra en la figura 4.

Figura 4: Razones trigonométricas

Con estos tres lados se pueden formar la seis razones siguientes:

b

c;

a

c;

b

a;

c

b;

c

a;

a

b

Se ene que estas seis razones dependen sólo del ángulo � y no del triángulo rectángulo formado. Enefecto, si se forma otro triángulo con hipotenusa de longitud c 0 y catetos de longitudes a0 y b0, entonceslos dos triángulos son semejantes y, por tanto, las razones correspondientes son iguales (ver figura 5).

Figura 5: Razones trigonométricas

Como resultado se ob ene:

b

c=

b0

c 0;

a

c=

a0

c 0;

b

a=

b0

a0;

c

b=

c 0

b0;

c

a=

c 0

a0;

a

b=

a0

b0

Como las razones dependen sólo del ángulo � y no del triángulo rectángulo formado, se da a cada razónun nombre especial que involucra al ángulo �:

sin � =b

c; cos � =

a

c; tan � =

b

a; csc � =

c

b; sec � =

c

a; cot � =

a

b

10

Page 11: M Dosier M3 Trigonometria

2.3. El círculo unitario

Considere el plano cartesiano xy . Sea � un círculo cuyo centro está en origen O del plano cartesiano yene radio igual a 1. Este círculo es conocido como El Círculo Unitario y la ecuación de la circunferencia

es x2 + y 2 = 1.Sea P un punto sobre la circunferencia. Las proyecciones de P sobre el eje x y el eje y son respec va-mente los puntos A y B. Por P se traza una tangente a la circunferencia, que corta al eje x y al eje y enlos puntos C y D, respec vamente. Finalmente, sea � el ángulo orientado que se forma desde el eje xposi vo hasta OP .

1. Si se supone que � es un ángulo entre 0� y 90�, entonces de la definición de las razones trigono-métricas se ob ene que AP = cos�, BP = sen�, CP = tan�, DP = cot�, OC = sec�,OD = csc�.

2. Si ahora suponemos que P es un punto cualquiera de la circunferencia y � es un ángulo que tomacualquier valor entre 0� y 360�, entonces se pueden generalizar las razones trigonométricas dela siguiente forma: los puntos de la figura enen por coordenadas P (cos�; sen�), A(cos�; 0),B(0; sen�), C(sec�; 0),D(0; csc�); así

La función seno de � viene dada por la coordenada en y del punto P .

La función coseno de � viene dada por la coordenada en x del punto P .

La función tangente de � se define como el cociente sen�cos�

, y su magnitud coincide con lalongitud del segmento tangente a la circunferencia PC.

La función cotangente de � se define como el cociente cos�sen�

, y su magnitud coincide con lalongitud del segmento tangente a la circunferencia PD.

11

Page 12: M Dosier M3 Trigonometria

La función secante de � viene dada por la coordenada en x del punto C, y su magnitudcoincide con la longitud del segmento secante a la circunferencia CO.

La función cosecante de � viene dada por la coordenada en y del punto D, y su magnitudcoincide con la longitud del segmento secante a la circunferenciaDO.

A par r de estas definiciones, se puede analizar el comportamiento de las funciones trigonomé-tricas cuando � varía de 0� a 360�, o equivalentemente en radianes de 0 a 2�. Para ello completela tabla siguiente (coloque “ND” si la función no está definida):

0

(0;

2

)�

2

(�

2; �

)�

(�;

3�

2

)3�

2

(3�

2; 2�

)sen� 0 Crece de 0 a 1 1 Decrece de 1 a 0 0 Decrece de 0 a�1 �1 Crece de�1 a 0cos�

tan�

cot�

sec�

csc�

3. Observe que todas las funciones trigonométricas f (�) cumplen f (�) = f (� + 2�), por lo queson funciones periódicas. Determine el periódo de cada una de ellas.

4. Observe que para todo �:

a) �1 � sen� � 1.

b) �1 � cos� � 1.

c) �1 < tan� <1.

d) �1 < cot� <1.

e) �1 � sec�, o bien sec� � 1.

f ) �1 � csc�, o bien csc� � 1.

5. Aplicando el Teorema de Pitágoras a los triángulos4OAP ,4OCP ,4ODP , se ob ene que paratodo � se cumple:2

a) sen2 �+ cos2 � = 1.

b) 1 + tan2 � = sec2 �.

c) 1 + cot2 � = csc2 �.

2.4. Razones trigonométricas para ángulos destacados

La siguiente tabla resume algunos valores del seno, coseno y tangente de ángulos agudos calculados apar r de figuras geométricas conocidas:

2Siempre que � permita que la función trigonométrica que se está evaluando esté bien definida.

12

Page 13: M Dosier M3 Trigonometria

a

O A

P

1

�4

�4

a

a

(a) Para �

4

O IH

P

1 1

h

�3

�3

�6

�6

H01H0

(b) Para �

3y �

6

Figura 6: Cálculo de seno y coseno para ángulos básicos

� 0�

6

4

3

2

cos� 1p3

2

p2

2

1

20

sen� 0 1

2

p2

2

p3

21

tan� 0p3

31

p3 ND

6

4

3

0

2

Es importante saber cómo se ob enen los valores de estos ángulos. Para 0 y �2, los valores son obtenidos

directamente de los valores de las coordenadas x e y de los puntos sobre la circunferencia que los des-criben. Para �

4(fig. 6a), notamos que el triángulo rectángulo es isósceles (ambos ángulos agudos deben

sumar �2) y por el teorema de pitágoras, a = 1p

2ó, equivalentemente, a =

p22. Ya que a corresponde a

las coordenadas x e y del punto P (asumiendo que O se encuentra en el origen), también correspondeal seno y coseno de �

4.Para encontrar el seno y coseno de �

3y �

6notamos que los ángulos son comple-

mentarios y que el triángulo rectángulo que los con ene es la mitad de un triángulo equilátero (ver fig.6b). Por geometría sabemos que la altura en un triángulo equilátero corresponde también a la bisectrizy mediatriz. Por lo tanto, ya que OP = 1, sabemos que OH = 1

2(por ser PH mediatriz), de donde el

cos �3= sen �

6= 1

2. Luego, por pitágoras, la altura PH =

p32, de donde sen �

3= cos �

6=

p32.

2.5. Iden dades Trigonométricas

Una Iden dad es una igualdad que es válida para todos los valores que pueda tomar la variable (o lasvariables, si hay más de una).3 En Trigonometría hay muchas iden dades y en esta sección obtendremosalgunas de las más conocidas.

2.5.1. Las iden dades básicas

3En cambio, una Ecuación es una igualdad que sólo es válida para ciertos valores de la variable, llamados Soluciones dela ecuación, y la variable toma el nombre de incógnita.

13

Page 14: M Dosier M3 Trigonometria

sin�

cos�0�

�2

��2

��

� � �

� + �

Figura 7

Para obtener iden dades trigonométricas a par r de nuestradefinición de seno y coseno de un ángulo dentro del círculounitario, primero observamos que los puntos de dicho círcu-lo asociados a �, � � �, � + � y �� son los vér ces deun rectángulo, suponiendo que los ejes de coordenadas sonlos ejes de simetría, como se muestra en la figura 7. De estaobservación, podemos hacer una lectura directa de la figurapara obtener las siguientes iden dades:

sin�

cos�0�

�2

�� � �

�2� ��

2+ �

Figura 8

cos(��) = cos� sen(��) = � sen�

cos(� � �) = � cos� sen(� � �) = sen�

cos(� + �) = � cos� sen(� + �) = � sen�

Si ahora se realiza la construcción para obtener ángulos com-plementarios dentro de la circunferencia, como se muestraen la figura 8, por congruencia de triángulos obtenemos lassiguientes iden dades:

cos(�2� �

)= sen� cos

(�2+ �

)= � sen�

sen(�2� �

)= cos� sen

(�2+ �

)= cos�

2.5.2. Iden dades de suma y resta de ángulos

Las siguientes iden dades a obtener serán aquellas que nospermitan obtener el valor del seno y coseno de la suma�+�y de la diferencia �� � en función del seno y coseno de � yde�. Encontraremos primero una iden dad para sen(�+�).

O

P

A

Q

C

B D

E

I

Figura 9

Sea P el punto sobre el círculo unitario que describe al ángulo �a par r del eje de coordenadas x , y Q el que describe al ángulo� a par r del punto P , como se muestra en la figura 9. Sea C elpunto donde cae la perpendicular a OP que pasa por Q. SeanB,D y A los puntos enOI donde caen las perpendiculares aOIquepasanpor los puntosQ,C yP , respec vamente. Finalmente,sea EC paralela a OI. Los siguiente datos pueden ser extraidosde esta construcción:

OQ = 1.

QC = sen� yOC = cos�.

Ya que DCEB es un palalelogramo, EB = CD.

sen� = CDOC

= CDcos�

, de donde CD = sen� cos� = EB.

14

Page 15: M Dosier M3 Trigonometria

Note que para obtener el valor del sen(� + �), es necesario conocer el valor de QB. De los datosanteriores obtuvimos el valor de EB, por lo que resta únicamente el valor de QE. Para esto, note quepor ángulos alternos internos entre paralelas \ECO = �. Pero \QCE + \ECO = �

2y también

\EQC + \QCE = �2, de donde \CQE = \ECO = �. Sabiendo esto, tenemos que cos� = QE

QC=

QEsen�

, de dondeQE = cos� sen�. Así,

sen(�+ �) = QB = EB +QE = sen� cos� + cos� sen�

A par r de esta iden dad pueden deducirse las mencionadas en el siguiente teorema, cuyas demostra-ciones se dejan al lector en forma de ejercicios.

Teorema 2.1. Para cualquier pareja de ángulos � y � se cumple que

sen(�+ �) = sen� cos� + cos� sen�sen(�� �) = sen� cos� � cos� sen�cos(�+ �) = cos� cos� � sen� sen�cos(�� �) = cos� cos� + sen� sen�

Demostración.

a) sen(�� �) = sen� cos� � cos� sen�

Deduciremos el resultado u lizando áreas. Veamos la siguiente figura:

A B

C

D

A B

D

A

D

C

a

basen�

bcos�

b

acos�

bsen�

a

b

�� �

� =

Tenemos que: (ACD) = (ACB)� (ADB). Calculado el área de cada triángulo:

(ACD) =a(b sen(�� �))

2

(ACB) =(b cos�)(a sen�)

2

(ADB) =(a cos�)(b sen�)

2

Sus tuyendo en la primera igualdad obtenemos el resultado deseado:

sen(�� �) = sen� cos� cos� sen�

15

Page 16: M Dosier M3 Trigonometria

b) cos(�� �) = cos� cos� + sen� sen�

Procederemos como en el literal anterior u lizando áreas.

AB

C

D B D

C

D

C

A

bcos�

acos�

aasen�

bsen�

bab

�2� (�� �)

+=

En la figura tenemos que: (BCA) = (BCD) + (DCA), calculado el área de cada triángulo:

(BCA) =a(b sen(�

2� (�� �)))

2=

a(b cos(�� �))

2

(ACB) =(a cos�)(b cos�)

2

(ADB) =(b sen�)(a sen�)

2

Sus tuyendo en la primera igualdad obtenemos el resultado buscado.

El siguiente ejemplo muestra una aplicación de las iden dades anteriores.

EJEMPLO 2.1Calcule el valor del seno y el coseno para el ángulo �

12.

Solución: Note que �3� �

4= �

12, por lo que podemos aplicar las iden dades para la diferencia de dos

ángulos y los valores ya conocidos para el seno y coseno de �4y �

3. Para el valor del coseno:

cos�

12= cos

(�3� �

4

)= cos

3cos

4+ sen

3sen

4

=1

2�p2

2+

p3

2�p2

2

=

p2

4(p3 + 1)

De manera similar obtenemos que sen �12

= sen(�3� �

4

)=

p24

(p3� 1

).

16

Page 17: M Dosier M3 Trigonometria

2.5.3. Iden dades de duplicación y linearización

De las fórmulas obtenidas en la sección anterior, tenemos el caso especial cuando � = �, de donde:

cos(2�) = cos2 �� sen2 � y sen(2�) = 2 sen� cos�

Si u lizamos la relación cos2 �+ sen2 � = 1 obtenemos:

cos(2�) = 2 cos2 �� 1 y cos(2�) = 1� 2 sen2 �

De donde es posible expresar al cos2 � y al sen2 � en función de cos(2�):

cos2 � =1 + cos(2�)

2y sen2 � =

1� cos(2�)

2

También podemos demostralas geométricamente u lizando la siguiente figura:

En el círculo unitario se construye el triángulo ABC con AB diámetro y C un punto cualquiera sobrela circunferencia. Trazamos la altura respecto a AB y así los triángulos ABC y ACD son semejantes(criterio A-A). Planteando las proporciones entre sus lados homólogos tenemos:

CD

CB=

AC

AB=

AD

AC

Si tomamos

CD

CB=

AC

ABsen(2�)

2 sen�=

2cos�

2sen(2�) = 2 sen� cos�

Y si tomamos

AC

AB=

AD

AC2 cos�

2=

1 + cos(2�)

2 cos�cos(2�) = 2 cos2 �� 1

17

Page 18: M Dosier M3 Trigonometria

EJEMPLO 2.2Calcule el seno y coseno de �

8.

Solución. Ya que conocemos el valor del coseno de 2 � �

8=

4, podemos u lizar las iden dades de

linearización con � = �8. Ya que cos �

4=

p22:

cos2�

8=

1 +p22

2=

2 +p2

4y sen2

8=

1�p22

2=

2�p2

4

La posición en el círculo unitario del punto asociada con �8muestra que cos �

8y sen �

8son números

posi vos. En consecuencia,

cos�

8=

√2 +

p2

2y sen

8=

√2�p2

2

2.6. Tabla resumen de iden dades trigonométricas más importantes

sen2 �+ cos2 � = 1 Iden dad Pitagórica1 + tan2 � = sec2 � ”1 + cot2 � = csc2 � ”

sen(�+ �) = sen� cos� + cos� sen� Iden dad del Seno de una Sumasen(�� �) = sen� cos� � cos� sen� ”cos(�+ �) = cos� cos� � sen� sen� Iden dad del Coseno de una Sumacos(�� �) = cos� cos� + sen� sen� ”

tan (�+ �) =tan�+ tan�

1� tan� tan�Iden dad de la Tangente de una Suma

tan (�� �) =tan�� tan�

1 + tan� tan�”

sen� sen� =cos (�� �)� cos (�+ �)

2Iden dad del Producto de Senos

cos� cos� =cos (�+ �) + cos (�� �)

2Iden dad del Producto de Cosenos

sen� cos� =sen (�+ �) + sen (�� �)

2Iden dad del Producto Seno-Coseno

sen(2�) = 2 sen� cos� Iden dad de Ángulo Doblecos(2�) = cos2 �� sen2 � = 1� 2 sen2 � = 2cos2 �� 1 ”

sen2( 2

)=

1� cos

2Iden dad del Ángulo Mitad

cos2( 2

)=

1 + cos

2”

senA+ senB = 2 sen

(A+ B

2

)cos

(A� B

2

)Iden dad de la Suma de Senos

senA� senB = 2cos

(A+ B

2

)sen

(A� B

2

)Iden dad de la Resta de Senos

cosA+ cosB = 2cos

(A+ B

2

)cos

(A� B

2

)Iden dad de la Suma de Cosenos

cosA� cosB = �2 sen(A+ B

2

)sen

(A� B

2

)Iden dad de la Resta de Cosenos

18

Page 19: M Dosier M3 Trigonometria

2.7. Lecturas y ejercicios complementarios

2.2.1.- MINED(2014). Lección 5: Trigonometría. En Material de Autoformación e Innovación Docentepara Matemá ca 1º y 2º año de Bachillerato. (pp.70-82). Viceministerio de Ciencia y Tecnología:El Salvador.

2.8. Problemas

PROBLEMA 2.1Ahora considere que el punto P se rota con centro el origen 90�, 180� y 270�, generando los puntosQ,R, y S, respec vamente. Por otra parte, las reflexiones del punto P con respecto al eje x y al eje y sonrespec vamente U y T . Finalmente, se construye el punto V que es la reflexión de P con respecto a larecta y = x .

1. Calcule las coordenadas de Q, R, S, T , U y V en términos de sen� y cos�. (U lice congruenciade triángulos.)

2. Demuestre que si � + � = 180� (suplementarios) entonces sen� = sen�, cos� = � cos� ytan� = � tan�.

3. Si en cambio, si � + � = 90� (complementarios) entonces sen� = cos�, cos� = sen� ytan� = cot�.

4. Mientras que si � = �� entonces sen� = � sen�, cos� = cos� y tan� = � tan�.

5. Determine los valores de seno y coseno de los ángulos de las forma siguiente (k 2 Z): 2k�, k�2

,

k�

6,k�

4,k�

3.

19

Page 20: M Dosier M3 Trigonometria

6. Resuelva sen (3x) =1

2, cos (4x) =

�p3

2.

7. Complete la siguiente tabla en términos de sen� y cos�

� �+�

2�+ � �+

3�

2� � � ��

2� �

sen� sen(�+

2

)= cos� sen (�+ �) = � sen� sen

(�+

3�

2

)= � cos� sen (� � �) = sen� sen (��) = � sen� sen

(�

2� �

)= cos�

cos� cos(�+

2

)= cos (�+ �) = cos

(�+

3�

2

)= cos (� � �) = cos (��) = cos

(�

2� �

)=

tan� tan(�+

2

)= tan (�+ �) = tan

(�+

3�

2

)= tan (� � �) = tan (��) = tan

(�

2� �

)=

cot� cot(�+

2

)= cot (�+ �) = cot

(�+

3�

2

)= cot (� � �) = cot (��) = cot

(�

2� �

)=

sec� sec(�+

2

)= sec (�+ �) = sec

(�+

3�

2

)= sec (� � �) = sec (��) = sec

(�

2� �

)=

csc� csc(�+

2

)= csc (�+ �) = csc

(�+

3�

2

)= csc (� � �) = csc (��) = csc

(�

2� �

)=

PROBLEMA 2.2Sean P yQ dos puntos sobre la circunferencia. Sean A y B las proyecciones respec vas de P yQ sobreel eje x . Sea C la proyección deQ sobreOP , mientras queD y E son las proyecciones de C sobre el ejex yQB, respec vamente.

Si \AOP = � y \POQ = �, demuestre que

1. OC = cos� y CQ = sen� .

2. CD = sen�cos� = EB .

3. \BOC = \BQC.

4. QE = cos � sen� .

5. QB = sen(�+ �) = sen�cos �+ cos� sen� .

.�sen�+ sen�cos�) = cos���cos(y

�sen�sen�cos�) = cos�+�cos(, demuestre que)

x2

�(

= senxcosU lizando la relación7.

.�sen�cos

��cos�) = sen���sen(, deduzca la fórmula))��+ (�) = sen (���sen(Dado que6.

20

Page 21: M Dosier M3 Trigonometria

1. Aprendizaje inicial de la Lengua es

Resolución de triángulos y

ecuaciones trigonométricas

21

Page 22: M Dosier M3 Trigonometria

Para la discusión de esta sección, usaremos las siguientes notaciones y convenciones para un triánguloABC dado:

a, b, c : las longitudes de los lados opuestos a los vér cesA, B y C, respec vamente.

S: el área del triángulo

cos A y sen A: el cos� y sen�, donde � es la medidaentre 0 y � de \BAC. A

C

b

ÂB

a

c

Ĉ

B

Teorema 3.1. (Ley del Coseno). En un triángulo ABC cualquiera, se cumple:

c2 = a2 + b2 � 2ab cos �

Demostración. Dado el triánguloABC, trazamos una circunferenciade radio a y centro en B. Tomamos el diámetro CF y un punto Gcualquiera sobre la circunferencia para formar el triángulo rectáguloCFG. Tenemos entonces que CG = 2a cos � y GA = 2a cos �� b.Ya que las cuerdas CG yDE son secantes tenemos que:

CA � AG = DA � AEb � (2a cos � � b) = (a � c) � (a + c)

2ab cos � � b2 = a2 � c2

c2 = a2 + b2 � 2ab cos �

a

c

2acos‘� b

a� c

b

a

BC

D

E

F

G

A

Teorema 3.2. (Ley de los Senos). En un triángulo ABC cualquiera, se cumple:

a

sen A=

b

sen B=

c

sen C=

abc

2S

3.1. Ecuaciones trigonométricas

Como se mencionó anteriormente, a diferencia de las iden dades, que se cumplen para cualquier valorde las variables en cues ón, las igualdades en las ecuaciones se cumplen sólo para algunos valores delas variables y resolverlas implica encontrar dichos valores. En ecuaciones trigonométricas, la principalestrategia es u lizar las iden dades antes vistas para simplificar lo más posible la igualdad. Por ejemplo,para la siguiente ecuación:

2 sen(x)� 1 = 0

Al sumar 1 a ambos lados de la ecuación y luego dividir por 2 obtenemos:

sen(x) =1

2

22

Page 23: M Dosier M3 Trigonometria

O

�6

�6

Figura 10

De los valores obtenidos para sen(x) en la sección 2.4 sa-bemos que x = �

6. Sin embargo, al resolver una ecuación

deben obtenerse todos los valores de la variable que hacenque la igualdad sea verdadera. Para el caso del seno, que enel círculo trigonométrico equivale a la coordenada y del pun-to sobre el círculo que describe al ángulo, podemos observaren la figura 10 que x = � � �

6= 5�

6también cumple con la

igualdad4. Es decir, tenemos que

x =�

6ó x =

5�

6

Pero hay que recordar, además, lo discu do en la sección 1.2: si desde cualquier punto del círculo trigo-nométrico se realiza una rotación completa, se regresa al mismo punto de par da, por lo que el seno delángulo descrito originalmente y el del ángulo que se forma luego de la rotación son exactamente losmis-mos. Así, para la primera de nuestras soluciones, sen(�

6) = sen(�

6+2�), de donde x = �

6+2� = 13�

6es

otra solución a nuestra ecuación. Ya que podemos repe r este mismo proceso de manera indefinida (esdecir, podemos dar tantas rotaciones completas como nos plazca y siempre llegaremos al mismo puntode par da), podemos decir que a par r de la solución x = �

6se pueden obtener infinitas soluciones de la

forma x = �6+ 2�k , donde k es un número entero cualquiera. Aplicando esta misma idea a la segunda

de las soluciones obtenidas anteriormente, podemos decir ahora que las soluciones de la ecuación son:

x =�

6+ 2�k ó x =

5�

6+ 2�k

donde k es un número entero.Note que elmétodou lizado en el ejemplo anterior es similar al que se u lizaría para resolver ecuacionescon coseno o tangente y puede ser descrito en los siguientes pasos:

1. Llevar la ecuación a la forma sen(x) = a, cos(x) = a, ó tan(x) = a.

2. Encontrar las soluciones entre 0 y 2� de estas ecuaciones simples. Para esto puede u lizar lasigualdades:

sen(x) = sen(� � x)

cos(x) = cos(�x)tan(x) = tan(� + x)

3. U lizar la propiedad periódica de las funciones para que, a par r de una solución específica x0, sepueda obtener la solución general x = x0 + 2�k , con k entero.

A con nuación se presentan más ejemplos que u lizan esta estrategia para su solución.

EJEMPLO 3.1Resuelva en R la ecuación sen x = �

p32.

4Note que lo que aquí descrito es una aplicación de la igualdad sen� = sen(���). Si la discusión se hubiese basado enel coseno, nos hubiese sido ú l la iden dad cos� = cos(��).

23

Page 24: M Dosier M3 Trigonometria

Solución. Conocemos una solución par cular de esta ecuación: x0 = ��3. La ecuación se convierte así

en sen x = sen(��

3

), que ene como soluciones:

x = ��3+ 2k� y x =

4�

3+ 2k�

donde k 2 Z. Note que el segundo conjunto de soluciones se ob ene tomando en cuenta la iden dadsen� = sen

(�2� �

).

EJEMPLO 3.2Resuelva en R la ecuación cos x = sen �

5.

Solución. Llevaremos esta ecuación a la forma cos x = cos x0 gracias a la relación:

sen�

5= cos

(�2� �

5

)= cos

3�

10

Por consiguiente, resolvemos la ecuación cos x = cos 3�10, de donde tenemos la solución

x =3�

10+ 2k� (k 2 Z)

Note que dado que cos� = cos(��), también tenemos como soluciones

x = �3�

10+ 2k� (k 2 Z)

EJEMPLO 3.3Resuelva en [0; 2�] la ecuación cos 3x = sen x .Solución. Tenemos que sen x = cos

(�2� x

), de donde la ecuación propuesta es equivalente a cos 3x =

cos(�2� x

). Obtenemos por lo tanto:

3x =�

2� x + 2k� o 3x = x � �

2+ 2k�

o equivalentemente:x =

8+ k

2o x = ��

4+ k�

con k 2 Z. De estos valores, aquellos que pertenecen al intervalo [0; 2�] son:

De la primera relación: �8, �8+ �

2, �8+ �, �

8+ 3�

2. Es decir: �

8, 5�

8; 9�

8; 9�13.

De la segunda relación: � � �4, 2� � �

4; es decir, 3�

4, 7�

4.

EJEMPLO 3.4Resuelva en [0; 2�] la ecuación 2 sen x cos x =

p3 cos(x).

Solución. Podemos transformar la ecuación dada a 2 sen x cos x �p3cos(x) = 0, de donde

cos x(2 sen x �p3) = 0

24

Page 25: M Dosier M3 Trigonometria

Esto es cierto cuando cualquiera de los dos términos que se están mul plicando son iguales a 0, por loque analizamos estos casos por separado. Para cos x = 0, podemos observar que en intervalo dado losvalores que cumplen con la ecuación son x = �

2ó x = 3�

2. Para 2 sen x � p3 = 0, transformamos la

ecuación para tener sen(x) =p32

ó sen(x) = sen �3. Dentro del intervalo dado, el único otro valor que

cumple con esta condición es x = � � �3= 2�

3.

Combinando los dos casos anteriores, tenemos que las soluciones de la ecuación son �3, �2, 2�

3, 3�

2.

3.1.1. Ecuaciones de la forma A cos (x) + B sen (x) = C

Note que en todos los ejemplos anteriores se han resuelto ecuaciones de la forma

A cos (x) + B sen (x) = C

Por lo que ahora describiremos un método general para la solución de dichas ecuaciones. Para esto,suponga que un triángulo rectángulo ene por catetos A y B, los coeficientes de sen (x) y cos (x) enla ecuación dada; su hipotenusa será entonces

pA2 + B2. Dividimos toda la ecuación por este valor,

resultandonos

ApA2 + B2

cos (x) +Bp

A2 + B2sen (x) =

CpA2 + B2

:

Los coeficientes en esta nueva ecuación son coordenadas de un punto en la circunferencia unitaria y porlo tanto se pueden iden ficar con el seno y coseno del ángulo � asociado a dicho punto. Existen dosposibilidades, de acuerdo a qué coeficiente será considerado abcisa y cuál de ellos como la ordenada.Por ejemplo, si hacemos

cos� =Ap

A2 + B2

sen� =Bp

A2 + B2

la ecuación puede ser escrita en la forma:

cos (�) cos (x) + sen (�) sen (x) =Cp

A2 + B2

De donde las soluciones de la ecuación original se reducen a las soluciones de la ecuación

cos (�� x) =Cp

A2 + B2

Note que esto nos dice que para que la ecuación original tenga solución debe cumplirse

�1 � CpA2 + B2

� 1

Note que la selección del sen� y cos� fue completamente arbitraria. También pudo haberse establecidoque

sen� =Ap

A2 + B2

25

Page 26: M Dosier M3 Trigonometria

cos� =Bp

A2 + B2

En cuyo caso la ecuación original se transforma en

sen (�) cos (x) + cos (�) sen (x) = sen(�+ x) =Cp

A2 + B2

EJEMPLO 3.5Resuelva en [0; 2�] la ecuación� cos (x) + sen (x) = 1p

2.

Solución. Note que en esta ecuación, A = �1, B = 1 y C = 1p2. De aquí que

cos� =�1√

(�1)2 + (1)2=�1p2=�p2

2

sen� =1√

(�1)2 + (1)2=

1p2=

p2

2

Dentro de [0; 2�] se enen dos opciones para� en la segunda ecuación: �4y 3�

4. De éstas, sólo 3�

4cumple

con la primera ecuación, por lo que la ecuación original se convierte en:

cos

(3�

4� x

)=

1

2

de donde 3�4� x = �

3ó 3�

4� x = ��

3; es decir, x = 5�

12ó 13�

12.

3.2. Lecturas y ejercicios complementarios

2.2.1.- MINED(2014). Lección 6: Ecuaciones Trigonométricas. En Material de Autoformación e Innova-ción Docente paraMatemá ca 1º y 2º año de Bachillerato. (pp.83-104). Viceministerio de Cienciay Tecnología: El Salvador.

26

Page 27: M Dosier M3 Trigonometria

2. Aprendizaje de la lectura inicial

Transformaciones en el plano: isometrías,

traslaciones y rotaciones

27

Page 28: M Dosier M3 Trigonometria

Las transformaciones geométricas son aplicaciones del plano en el plano tal que a cada punto de unplano le hace corresponder otro punto del mismo plano. Las transformaciones geométricas permitendeducir una nueva figura de la primi vamente dada. El trasformado se le llama Homólogo del original.

Podemos clasificar las transformaciones en función del aspecto de la figura homóloga respecto a la ori-ginal en:

Isometrías: Un movimiento o isometría es una transformación que preserva todas las distanciasy por ello preserva el tamano y la forma (dimensiones y los ángulos). Estudiaremos la simetríacentral y axial, traslaciones y rotaciones.

Isomórficas: Estas trasformaciones conservan la forma de la figura original (los ángulos) y existeproporcionalidad entre las dimensiones de la figura original y la homóloga. Estudiaremos la ho-motecia.

4.1. Isometrías: Simetrías.

Estudiaremos en este apartado dos pos de simetrías, las centrales y las axiales. En las primeras la si-metría ocurre respecto a un punto denominado centro, mientras que en las segundas la simetría se darespecto a una recta que es denominada eje de simetría.

4.1.1. Simetrías Centrales

Considere los puntos del plano P y un puntoC que denominaremos centro. La simetría central de centroC es una transformación del plano que envía cada punto P del plano en otro punto P 0 del plano demanera tal que el punto C es el punto medio del segmento PP 0, es decir, PC = CP 0.

P C P 0

Vamos aplicar esta transformación a varios elementos geométricos. Iniciemos por un segmento. Al seg-mento PQ le apliacremos la simetría respecto al punto C, para ello basta aplicarla a los extremos y unirlos extremos resultantes.

P Q

C

P 0Q0

Podemos notar que el nuevo segmento Q0P 0 es congruente con el segmento PQ, ya que los4PCQ y4P 0CQ0 son congruentes (se ene queCP = CP 0 yCQ = CQ0 y la igualdad de ángulos[PCQ = \P 0CQ0

criterio LAL).

Veamos que sucede con un triángulo.Haremos la simetría del triángulo ABC respecto al puntoD. Apli-camos la transformación a cada uno de los vér ces y obtenemos el nuevo triángulo A0B0C 0

28

Page 29: M Dosier M3 Trigonometria

Es fácil probar que los triángulos ABC y A0B0C 0 son congruentes ya que enen sus tres lados iguales.Por ejemplo para probar queAB = A0B0 observemos que los triángulosABD yDAB0 son congruentes,por tener los lados BD = DB0 y AD = DA0 y los ángulos [ADB = \A0DB0, por tanto AB = A0B0. Demanera análoga podemos demostrar la igualdad de los otros lados.

4.1.2. Propiedades de las simetrías centrales.

Las simetrías centrales enen la propiedad de conservar:

1. Distancias, es decir la distancia entre dos puntos y sus correspondientes imágenes es la misma, esdecir d(P;Q) = d(P 0; Q0):

2. Puntos medios. Esta propiedad significa que dados dos puntos P yQ y sus respec vas imágenes P 0 yQ0, el punto medio M del segmento [P;Q] ene por imagen aM 0 que es el punto medio de [P 0; Q0].

3. El alineamiento de puntos. Esto significa que si tres omás puntos están alineados, sus imágenes estánigualmente alineadas.

4. Los ángulos y su orientación. Es decir, si [PQR = �, entonces \P 0Q0R0 = �

5. Las áreas. Es decir si una figuraF ene áreaA, la imagenF 0 que se ob ene después de la simetrizaciónes siempre igual a A:

Demostración. 1. Tomando en cuenta que la distancia de d(C; P ) = d(C; P 0) y d(C;Q) = d(C;Q0) yla igualdad de ángulos [PCQ = \P 0CQ0, por el criterio LAL de congruencia de triángulos se deduceque los triángulos4PCQ y4P 0CQ0 son congruentes y en consecuencia d(P;Q) = d(P 0; Q0).

P Q

C

P 0Q0

2. Como en la prueba anterior, los triángulos4PCM y4MCQ son respec vamente congruentes conlos triángulos4P 0CM 0 y4M 0CQ0, respec vamente; en consecuenciaP 0M 0 = PM = MQ = M 0Q0,de donde se deduce que P 0M 0 = M 0Q0, es decirM 0 es el punto medio de P 0Q0.

29

Page 30: M Dosier M3 Trigonometria

P Q

C

P 0Q0

M

M 0

3. Sean P , Q y R tres puntos alineados, condición que significa que PQ + QR = PR. Por la primerade las propiedades de las simetrías centrales, ya demostrada, de conservar distancias, sabemos queP 0Q0 = PQ, Q0R0 = QR y P 0R0 = PR, en consecuencia se ene: P 0Q0 + Q0R0 = PQ + QR =

PR = P 0R0 y en consecuencia P 0; Q0yR0 están alineados

4. Considere el ángulo [PQR. Verifique que el triángulo PQR es congruente con el triángulo P 0Q0R0 yque en consecuencia sus ángulos correspondientes son iguales, en par cular tendremos la igualdad[PQR = \P 0Q0R0.

Además, las simetrías centrales ene la propiedad de que la composición con ella misma da como resul-tado la iden dad. Es decir si Sc es la simetría de centro c , entonces S2

c (P ) = I(P ) = P:; por otra partelas simetrías centrales ene la propiedad de que el segmento[P;Q] y el segmento imagen [P 0; Q0] sonparalelos.

Se dice que un punto P es invariante por una transformación cuando su imagen P 0 coincide con P . Sedice también que el punto es un punto fijo por la transformación. En el caso de las simetrías centrales elúnico punto que permanece invariante por ella es el centro mismo de la transformación; para cualquierotro punto P del plano se ene que su imagen P 0 es diferente a P .

4.1.3. Simetrías Ortogonales

Sea D una recta del plano. Llamamos simetría axial ortogonal o reflexión a la transformación del planoque a cada punto P del plano lo transforma en el punto P 0 que sa sface las siguientes condiciones:

1. El punto medio del segmento PP 0 pertenece a la rectaD:

2. El segmento PP 0 es ortogonal o perpendicular a la rectaD:

D

P

P 0

30

Page 31: M Dosier M3 Trigonometria

De la definición se puede concluir que la recta D es mediatriz del segmento PP 0, por ello si tomamosun punto cualquiera E sobre la rectaD se cumple que PE = P 0E.

Apliquemos esta transformación a un segmento cualquiera. Dado el segmento PQ lo reflejaremos res-pecto a la recta D. Se verifica que el segmento resultante P 0Q0 es de igual longitud que el segmentoPQ

Veamos que ocurre si reflejamos un triángulo cualquiera ABC respecto a la rectaD.

El triángulo resultante A0B0C 0 es congruente con el triángulo ABC

4.1.4. Propiedades de las simetrías ortogonales

Las propiedades son similares a las propiedades de las simetrías centrales, salvo que la propiedad deconservación de los ángulos se reduce sólo a la conservación de la medida de los ángulos, no así laorientación de los mismos que en las simetrías ortogonales la orientación se invierte. Todas las demáspropiedades son idén cas.

31

Page 32: M Dosier M3 Trigonometria

Una figura se dice que ene un eje de simetría ortogonal de eje D, cuando la figura queda invariante alaplicarle la simetría. De las figuras usuales en geometría las siguientes poseen eje de simetría:

1. Los rombos. Sus ejes de simetría son sus diagonales.

D

D’P R

S

Q

SD(P ) = R, SD(Q) = Q, SD(R) = P y SD(S) = S.

SD0(P ) = P , SD0(Q) = S, SD0(R) = R y SD0(S) = Q.

2. El triángulo4ABC isósceles en A ene como eje de simetría la altura que sale de A.

3. El triángulo equilátero ene sus tres alturas como ejes de simetría.

4. El rectángulo admite como ejes de simetría a las rectas que unen los puntos medios de sus ladosparalelos.

5. El cuadrado admite como ejes a sus diagonales y a las rectas que unen puntos medios de sus ladosparalelos.

P Q

RS

D2

D1

E1

E2

En este caso, por ejemplo, se enen las siguientes correspondencias:SD1

(Q) = Q, SD1(S) = S, SD1

(R) = P y SD1(P ) = R.

SE1(Q) = R, SE1

(S) = P , SE1(R) = Q y SE1

(P ) = S.

32

Page 33: M Dosier M3 Trigonometria

Las simetrías ortogonales además enen, como las simetrías centrales, la propiedad de que la compo-sición con ella misma es la iden dad; es decir que S2

D(P ) = I(P ) = P ; sin embargo, en las simetríasortogonales, un segmento [P;Q] y su segmento imagen [P 0; Q0], en general, no son paralelos.

En las simetrías ortogonales los puntos fijos son todos los puntos que pertenecen a la recta de simetri-zación.

4.1.5. Algunas ac vidades para realizar en el aula

Ac vidad 1: Con esta ac vidad el alumno podrá descubrir que ocurre al reflejar un triángulo respecto auna recta.

1. Dobla una hoja de papel. Hazle tres perforaciones con un alfiler, marcando estas con las letras A,B y C y vuelve a desdoblarla.

2. UneA conA0 ( con línea punteada y fina ) ;(A0 es el punto imagen deA resultante de la perforacióndel alfiler );B con B0 y C con C 0.

3. Une A conB y C. Estas con línea entera. También une A0 conB0 y con C 0. Resultan dos triángulos.Colorea los triángulos resultantes

4. Mide el segmento desde A hasta la línea de doblez y desde esta hasta A0. Igual con B y C. ¿ quesucede ?

5. ¿Qué se puede decir del segmento AA0 con respecto al doblez?

Ac vidad 2: Con esta ac vidad se prentende que el alumno iden fique la simetría de algunas figuras.

1. Doble una hoja de papel tamano carta de tal manera que forme un cuadrado con la mayor áreaposible

2. Ahora, elimine la parte de papel que sobra recortándolo, dejando así sólo el cuadrado

3. Luego, doble el cuadrado juntando dos esquinas opuestas. Repita el procedimiento con las otrasdos esquinas opuestas. Marque bien ambos doblados y formará las diagonales del cuadrado

4. A con nuación, junte las diagonales de manera que forme un triángulo

5. Junte las diagonales del cuadrado en un lado de un triángulo.

6. Luego, con una jera, haga cortes en lados del triángulo que no sean los lados del cuadrado.

7. Tenga presente que los cortes serán duplicados dependiendo de los dobleces que haya realizado.

8. ¿Qué propiedad enen las figuras geométricas que observa en el papel producto de los cortes?

Ac vidad 3: Encuentre la imagen de cada figura aplicando simetría central o axial.

33

Page 34: M Dosier M3 Trigonometria

34

Page 35: M Dosier M3 Trigonometria

4.2. Traslaciones.

Una traslación desliza la figura a lo largo de una trayectoria recta, moviendo cada punto la misma dis-tancia en la misma dirección. Se puede describir una traslación usando un vector de traslación, queespecifica tanto la distancia como la dirección.

Definición: Sea �!u un vector, se llama traslación del vector �!u a la asociación de los puntosM yM 0 delplano tales que:

���!MM 0 = �!

u .

En general se denota t�!u a la traslación del vector�!u , y escribimos:M 0 = t�!uM oMt�!u�! M 0

Para ilustrar esta aplicación u lizaremos el plano cartesiano. Tenemos el triángulo CDE al cual le apli-caremos la traslación del vector ~u.

El vector ~u nos indica que relizaremos un desplazamiento de cinco unidades hacia la derecha y tresunidades hacia arriba. Si anclamos el vector u en el origen lo podemos representar u = (5; 4). Así porejemplo la imagen del punto C(2; 2) la encontramos desplazando este punto cinco unidades hacia laderecha y tres unidades hacia arriba obteniendo el punto C 0(7; 5), de manera análoga encontramos lasimagenes de los otros dos vér ces:

t~u(C(2; 2)) = C 0(7; 5)

t~u(D(4; 6)) = D0(9; 9)

t~u(E(9; 2)) = E 0(14; 5)

35

Page 36: M Dosier M3 Trigonometria

4.2.1. Ac vidad

Traslade cada figura según el vector dado.

4.2.2. Propiedades de las Traslaciones.

Las traslaciones poseen las siguientes propiedades:

La imagen de una recta, es otra recta paralela a ella.

La imagen de un segmento, es otro segmento de igual longitud.

La imagen de una circunferencia de centroO, es otra circunferencia del mismo radio, cuyo centroes la imagen de O por la traslación.

Conservación del alineamiento de puntos.

36

Page 37: M Dosier M3 Trigonometria

Dado que la imagen de una recta es otra recta, también se cumple que si tres puntosA;B; C estánalineados, las imágenes respec vas A0; B0; C 0 están también alineados.

Conservación del paralelismo.

Si d1 y d2 son dos rectas paralelas, las imágenes respec vas d 01 y d02 son paralelas también.

De esta propiedad es fácil deducir que: “La imagen de un paralelogramo, es otro paralelogramo ”.

Conservación de las distancias y las áreas.

La imagen de un segmento, es también un segmento de la misma longitud.

La imagenD0 de una superficieD, ene la misma área queD.

Conservación del punto medio de un segmento.

Sea I el punto medio del segmento AB, si A0B0 es la imagen del segmento AB, entonces I 0 es laimagen de I, además I 0 es el punto medio del segmento A0B0.

Conservación de la medida de ángulo y de la ortogonalidad.

La imagen\x 0A0y 0 del ángulo xAy ene la misma medida que xAy .

En par cular cuando dos rectas d1 y d2 son perpendiculares sus respec vas imágnes d 01 y d 02 sontambién perpendiculares.

EJEMPLO 4.1A, B, C son tres puntos tales que C es la imagen de B por la traslación de

�!AB. Haga una figura de esta

situación y responda ¿Cuál es el punto medio de AC?

Demostración. �!u =�!AB luego por definición tu(A) = B, y de acuerdo a las propiedades de las trasla-

ciones la imagen de AB es otro segmento de la misma longitud, y como AB = BC, entonces B es elpunto medio de AC.

A Bu C

EJEMPLO 4.24ABC es un triángulo cualquiera. Denotaremos por B0 a la imagen de B por la simetría central decentro A. Trazamos la recta d paralela a AB y que pasa por C, y la recta � paralela a BC y que pasapor A. Las rectas d y � se cortan en C 0. Demostrar que el triángulo4B0AC 0 es la imagen del triángulo4ABC por traslación.

Demostración. De acuerdo a la forma del4ABC (Ver figura), transformamos el4ABC en el4B0AC 0

por la traslación del vector�!BA hacia

��!A0B0 o

��!CC 0. Por lo cual, será suficiente demostrar que tu(A) = B0,

tu(B) = A y tu(C) = C 0.37

Page 38: M Dosier M3 Trigonometria

tu(B) = A, este resultado se ob ene de la definición de traslación del vector�!u .

tu(A) = B0, en efecto, B0 es la imagen de B, por la simetría central de centro A. Como A esel punto medio de BB0, se ene que

�!BA =

��!AB0. Es decir

��!AB0 = �!

u y esto por definición estu(A) = B0.

tu(C) = C 0, en efecto, los lados opuestos del cuadriláteroAC 0CB son dos a dos paralelos, es decirse trata de un paralelogramo, esto es:

��!CC 0 =

�!BA =) ��!

CC 0 = �!u , lo cual significa que tu(C) = C 0.

A

B C

d

C 0

B0

EJEMPLO 4.3Dado el 4ABC sea H su ortocentro. BCDE es un rectángulo construido exteriormente. Se trazan,por D la perpendicular a AB, y por E la perpendicular a AC. Estas dos rectas se cortan en I. U lice latraslación t�!u del vector�!u =

�!EB para demostrar que A, H e I están alineados.

Demostración. Dado �!u =�!EB, por definición tu(E) = B, y dado que BCDE es un rectángulo tam-

bién se cumple tu(D) = C. Como la imagen por una traslación de una recta es otra recta paralela a laprimera, la recta BM es la imagen por�!u de la recta ER, dado que al ser ambas perpendiculares a ACresulta inmediato que ER k BM. Análogamente, la imagen de la rectaDS por�!u es la recta CP .

La intersección de ER y DS es I, como la traslación es biyec va (en el sen do de que es punto a pun-to) ésta lleva intersecciones de rectas a intersecciones de rectas, luego t�!u (I) = H y esto implica queIH ? BC (dado que

�!IH = �!

u por definición de traslación, y este úl mo vector evidentemente es per-pendicular a BC dada la forma cómo se definió). Como H es el ortocentro del4ABC, AH ? BC. AsíIH k AH (ambas son perpendiculares a BC) y esto obliga a que A, H, I estén alineados.

A

B C

DE

RS

MP

I

H

38

Page 39: M Dosier M3 Trigonometria

4.3. Rotaciones.

4.3.1. Rotación en el plano orientado.

Un ejemplo es ú l ahora para entender la importancia de trabajar en el plano orientado. En este casoel centro de rotación es O y el ángulo es �

2. En la figura adjunta, además de la ortogonalidad indicada,

suponga que, OM2 = OM1 = OM. La formulación: “Un puntoM asociado aM 0 talque OM 0 = OM y\MOM 0 = �

2” es ambigua, dado que los puntosM1 yM2 cumplen con la condición.

M

OM2 M1

En cambio, en el plano orientado, la formulación: “Un punto M asociado a M 0 tal que OM 0 = OM y(��!OM;

��!OM 0) = �

2” no deja ninguna duda: un solo punto cumple con la condición,M 0 = M1. Por lo que

en adelante, se u lizará el plano orientado.

Definición 4.1. M 0 es la imagen deM (M 6= O) por la rotación de centro O y ángulo � si: OM 0 = OM

y (��!OM;

��!OM 0) = �.

O M

M 0

Notación:R�O es la rotación de centroO y ángulo �.

4.3.2. Ac vidad

Rota cada figura respecto al punto indicado y el ángulo dado.

39

Page 40: M Dosier M3 Trigonometria

� = 75 � = 60

� = 45 � = 90

� = 120 � = 145

cccc

Puntos Invariantes:

Si � = 0 entonces todos los puntos son invariantes.

Si � 6= 0 entonces el centroO de la rotación es el único punto invariante.

Biyec vidad y Rotación Inversa: R�O es biyec va y la rotación inversa es la rotación de centro O y de

ángulo��.40

Page 41: M Dosier M3 Trigonometria

Demostración. Basta demostrar que:R�O(M) = M 0 si y sólo siR��

O (M 0) = M.

4.3.3. Propiedades Esenciales

Teorema 4.1. La rotación conserva distancias y ángulos.

1. La rotación envía a un segmento AB a otro A0B0, igual en longitud, siendo A0 imagen de A, B0

imagen de B. Mas aún el segmento AB se transforma en el segmento A0B0.

2. La rotación transforma a una recta en otra, siendo el ángulo entre ellas �.

Demostración. Sea A0 = RO;�(A) y B0 = RO;�(B), entonces, OA = OA0, OB = OB0 y \A0OB0 =

\AOB0 � \AOA0 = \AOB0 � \BOB0 = \AOB, así por LAL el4A0OB0 es congruente con4AOB,por tanto, la longitud del segmento AB es la misma que la del segmento A0B0.A con nuación demostramos que la imagen del segmento AB es el segmento A0B0. Sea C un puntodel segmento AB o su prolongación, y C 0 = RO;�(C), se demuestra como antes que A0C 0 = AC

y B0C 0 = BC, por consiguiente A0C 0 + B0C 0 = AC + CB = AB = A0B0, dado que A, B y C estanalineados, por lo tanto,A0C 0+C 0B0 = A0B0 lo que demuestra queA0,B0 yC 0 alineados. En consecuencia,el segmento A0B0 es la imagen de AB y la recta A0B0 es la imagen de la recta AB.Ahora, determinamos el ángulo entre la recta y su imagen. Defina D como la intersección de las rectasAB y A0B0. Por la congruencia de los triángulos AOB y A0OB0, se ene que \BAO = \B0A0O y porende el cuadrilátero ADA0O es cíclico, por consiguiente, si E es un punto sobre la recta AD entonces,\EDA0 = \AOA0 = �.

A

B

O

A0

B0C

C 0

Corolario. Suponga que: A0 = RO;�(A), B0 = RO;�(B) C 0 = RO;�(C), demuestre que:

La imagen del punto medio de AB es el punto medio de A0B0.

La rotación transforma a un círculo de centro C y radio r en otro de igual radio y centro C 0.

Si AB es diámetro del círculo de centro C, entonces A0B0 es el diámetro del círculo de centro C 0.

La rotación transforma un triángulo ABC en otro igual A0B0C 0

La rotación transforma a un ángulo en otro igual e igualmente orientado.

Dos rectas ortogonales ene por imagen dos rectas ortogonales.41

Page 42: M Dosier M3 Trigonometria

Dos rectas paralelas enen por imagen dos rectas paralelas.

Demostración. Dejamos esta prueba como ejercicio.

EJEMPLO 4.4Considere dos triángulos rectángulos isósceles ABC y AB0C 0 como en la figura adjunta. Demuestre queBB0 = CC 0 y que las rectas BB0 y CC 0 son ortogonales.Solución. Dado que por hipótesis AB = AC, AB0 = AC 0 y el \BAC = \B0AC 0 = �

2, entonces,

C = RA; �2(B) y C 0 = RA; �

2(B0). Así aplicando las propiedades de conservación de distancias y ángulos

de las isome as se concluye que BB0 = CC 0 y como el ángulo de rotación es �2, BB0 ? CC 0.

A

B

C

B0

C 0

EJEMPLO 4.5U lización de una rotación Sea ABCD un cuadrado de centro O.M es un punto del segmento AB, yN es un punto del segmento AD tal que DN = AM. Considere la rotaci’on de centro O y de ángulo �

2

en sen do posi vo. Demuestre que OM = ON y el \MON = �2.

Solución. Recuerde que en todo cuadrado las diagonales son mediatrices mutuamente, así que OA =

OD y el\AOD = �2, en consecuenciaD = RO; �

2(A), de la misma forma se verifica que:A = RO; �

2(B),

así por las propiedades de conservación de la rotación la imagen del segmento AB se transforma en elsegmento DA, y por consiguiente M 0 = RO; �

2(M) es el punto del segmento DA tal que DM 0 = AM,

por lo tanto M 0 = N, por definición de rotación se ene que OM = OM 0 = ON y el \MOM 0 =

\MON = �2.

42

Page 43: M Dosier M3 Trigonometria

D C

BA

O

M

N

EJEMPLO 4.6En la figura adjunta, ABCD es un rombo y los triángulosDCI yBCJ son equiláteros. Defina una trans-formación geométrica y demuestre que A, I y J están alineados.Solución. La presencia de dos triángulos equiláteros del mismo vér ce C nos sugiere definir la rotaciónde centro C y ángulo �

3. Por esta rotación B es la imagen de J, D la imagen de I. Ahora definimos A0

como la imagen de A, por consiguiente, el triángulo CAA0 es equilátero. Esto demuestra que B, D yA0 están alineados, dado que pertenecen a la mediatriz de AC, puesto que: BA = BC, DA = DC yA0A = A0C. Ahora solo falta considerar la rotación inversa de centro C y ángulo��

3para demostrar que

J = RC;� �

3(B), I = RC;� �

3(D), A = RC;� �

3(B) están alineados las propiedades de conservación de

colinealidad de las isometrías.

DC

A B

J

I

EJEMPLO 4.7Dada una circunferencia � de centro C y radio CO. Demuestre que: Si � 0 = RO;�(� ), P 2 � , Q 2�∩� 0 y P 0 = RO;�(P ) entonces, P ,Q y P 0 están alineados.

Solución. Por propiedades de conservación de isometrías sabemos que � 0 es la circunferencia de centroC 0 = RO;�(C) y radio C 0O, ya que O es el punto invariante de la rotación. Además, dado que las rota-ciones conservan ángulos se ene que el\PCO = \P 0C 0O = 2�, entonces, el\OQP = 180�� � porser inscrito en la circunferencia � cuyo ángulo central es \PCO, y el \P 0QO = � por ser inscrito en lacircunferencia � 0 cuyo ángulo central es\P 0CO. Por consiguiente,\OQP +\P 0QO = 180���+� =

180� y por lo tanto, P ,Q y P 0 son colineales.43

Page 44: M Dosier M3 Trigonometria

1800 � �

2�

2�

C

O

P

P 0C 0

Q

4.4. Transformaciones en el plano: composición de simetrías y homotecias

En está sección:

Encontrarás la transformación simple equivalente a la composición de dos traslaciones.

Encontrarás la transformación simple equivalente a la composición de las reflexiones a través dedos rectas paralelas

Encontrarás la transformación simple equivalente a la composición de las reflexiones a través dedos rectas que se intersecan

Cuando se aplica una transformación a una figura y luego se aplica otra transformación a su imagen, latransformación resultante se llama una composición de transformaciones.

4.5. Ac vidad 1

Reflexiones a través de dos líneas paralelas. En primer lugar, consideremos el caso de líneas paralelaspara la reflexión

1. En una hoja de papel vegetal o un acetato, dibuja una figura y una línea de reflexión pero que nose corten.

2. Doble el papel por la línea para reflejar su figura, marque la figura resultante.

44

Page 45: M Dosier M3 Trigonometria

3. En el papel vegetal o en el acetato, dibujar una segunda recta de relexión paralela a la primera, talque la imagen se encuentre entre las dos rectas paralelas.

4. Refleja la imagen respecto a la segunda recta y traza la segunda imagen.

5. �ué transformación única llevaría la figura original a la imagenfinal? (Sugerencia: �ómo se comparala orientación de la imagen final con la orientación de la original?)

Compara la distancia entre las rectas paralelas con la distancia entre un punto en la figura original y elpunto correspondiente en la imagen final.Usa tus descubrimientos para completar esta conjetura:

45

Page 46: M Dosier M3 Trigonometria

4.6. Ac vidad 2

Reflexiones a través de dos rectas que se intersecan. A con nuación, se estudiará el caso de rectasrectas que se intersectan para la reflexiń.

1. En una hoja de papel vegetal o un acetato, dibuja una figura y una línea de reflexión pero que nose corten.

2. Dobla el papel por la línea para reflejar su figura, marque la figura resultante.

3. En el papel vegetal dibuja una segunda recta de reflexión que se corte con la primera, de maneratal que la imagen forme un ángulo agudo entre la intersección de las lineas de reflexión.

4. Refleja la imagen respecto a la segunda recta y traza la segunda imagen

5. Dibuja dos semirrectas p y q que comiencen en el punto de intersección de las dos rectas y que pa-sen por los puntos correspondientes de la figura original y su segunda imagen. �ué transformaciónsimple llevaría la figura original a la imagen final?

46

Page 47: M Dosier M3 Trigonometria

Debes haber encontrado que las dos reflexiones son equivalentes a una rotación simple. Usa un trans-portador para comparar el ángulo de rotación (es decir, el ángulo creado por las semirrectas p y q) conel ángulo agudo formado por las rectas que se intersecan.

Usa tus descubrimientos para completar esta conjetura:

4.7. Ac vidad 3

Ahora veremos un ejemplo de composición de dos traslaciones.El triángulo ABC con vér ces en los puntos A(�1; 0), B(4; 0) y C(2; 6), en primer lugar es trasladadopor el vector ~u = (�6; 5) y después su imagen, el triánguloA0B0C 0 es trasladadopor el vector~v = (6; 3).Qué traslación simple es equivalente a la composición de esas dos traslaciones?

47

Page 48: M Dosier M3 Trigonometria

Podemos observar que al componer las dos traslaciones resulta otra traslación cuyo vector es el ~w =~u + v

Ahora pasaremos a formalizar los resultados anteriores.

4.8. La isometría directa más simple

Teorema 4.2. Dados dos segmentos congruentesAB yA0B0, existe una isometría directa que transformaa A en A0 y a B en B0. Además, de todas las isometrías que cumplen esta condición, las más simplesiempre será una rotación, o una traslación en caso excepcional.

Demostración. Si ABB0A0 forma un paralelogramo, claramente la traslación de vector��!AA0 transforma

A en A0 y B en B0. Si en cambio ABA0B0 forma un paralelogramo, tomando O como la intersección deAA0 con BB0, una rotación de centro O y ángulo 180� funciona.5

A B

A0 B0

A B

B’ A’

O

5Observe que esta rotación es una reflexión puntual respecto a O.

48

Page 49: M Dosier M3 Trigonometria

En cualquier otro caso (es decir, AB , A0B0), sea P la intersección de AB con A0B0, y sea O la otraintersección de los circuncírculos de los triángulos4AA0P y4BB0P . Note que OPAA0 y OPBB0 soncuadriláteros concíclicos, entonces \OAB = \OA0B0 y \OBA = \OB0A0, y dado que por hipótesisAB = A0B0 se concluye que 4ABO � 4A0B0O y enen igual orientación. Note además que si lla-mamos � al ángulo que la recta AB forma al rotarse con centro P hasta coincidir con la recta A0B0, denuevo por los cuadriláteros concíclicos se ob ene � = \AOA0 = \BOB0. Así, una rotación de centroO y ángulo � funciona como isometría directa que transforma a A en A0 y a B en B0.

A B

A0

B0

P

O

4.9. Descomposición de traslaciones y rotaciones como composición de simetríasaxiales

Ya se estudió el hecho que una simetría axial es una isometría indirecta, esto quiere decir que a pesarque man ene invariante las distancias cambia la orientación de los ángulos. En par cular, si los vér cesde un triángulo 4ABC se recorren en sen do an horario, los vér ces del triángulo imagen tras unasimetría axial4A0B0C 0 se recorren en sen do horario. Si ahora se aplica otra simetría axial (o la mismareflexión axial anterior, si se quiere) al4A0B0C 0, el triángulo imagen4A00B00C 00 cumple que sus vér cesse leen en sen do an horario. Así, el4ABC y el4A00B00C 00 son triángulos congruentes y con la mismaorientación, esto significa que existe una isometría directa que transforma al4ABC en el4A00B00C 00.

Por otra parte, ya se sabe que una isometría directa siempre puede resumirse a una sola rotación, o unasola traslación en caso excepcional. A pesar que no es evidente, las Reflexiones Axiales pueden u lizarsepara generar Rotaciones y Traslaciones, lo que las convierte en las isometrías más fundamentales. Estosignifica que cualquier isometría puede escribirse como la composición de reflexiones axiales. Más im-presionante aún es que la can dad de reflexiones axiales necesarias es únicamente 2.

Teorema 4.3. Una Traslación puede descomponerse como el producto de dos reflexiones axiales.

Demostración. Si la traslación está determinada por el vector�!AB, al tomar dos rectas paralelas entre sí

l1 y l2, y perpendiculares a�!AB, tales que la distancia desde l1 hasta l2 es igual a la mitad de la longitud

del vector�!AB, se cumple que la composición de reflexiones axialesRl2 �Rl1 es igual a la traslación T�!AB.

49

Page 50: M Dosier M3 Trigonometria

l1 l2

2d

d

A B

P P1P2

Teorema 4.4. Una Rotación puede descomponerse como el producto de dos reflexiones axiales.

Demostración. Dada la rotación de centro O y ángulo �, se construyen dos rectas l1 y l2 que pasan porO, y que cumplen que el ángulo desde l1 hasta l2 es igual a �

2. Es rela vamente sencillo mostrar que la

composición de reflexiones axialesRl2 � Rl1 es igual a la rotación de centro O y ángulo �.

l1

l2

O

A

A1

A2

B

B1

B2

A par r de estos dos teoremas anteriores es evidente el siguiente corolario:

Corolario. Toda Isometría Directa puede descomponerse como el producto de 2 Reflexiones Axiales.

Más aún, una Isometría Indirecta o bien es una Reflexión Axial o bien la composición de una IsometríaDirecta con una Reflexión Axial, por lo tanto, toda Isometría Indirecta puede escribirse como la composi-ción de a lo sumo tres Reflexiones Axiales. De aquí se deriva el siguiente resultado, que es fundamentalen la teoría de Transformaciones Geométricas.

Teorema 4.5. Toda Isometría puede escribirse como la composición de a lo sumo 3 Reflexiones Axiales.

4.10. Problemas de Traslaciones

1. Sean ABCD un paralelogramo de centro O, la paralela a BD que pasa por A corta a la paralelaAC que pasa porD en el punto E. ¿Cuáles son las imágenes de A yD bajo la traslación del vector�!EO?

2. SeanABCD un paralelogramo de centroO,M yN los puntos medios de los segmentosDA yBCrespec vamente. ¿Son verdaderas o falsas las siguientes afirmaciones?

50

Page 51: M Dosier M3 Trigonometria

a) Los puntosM; O y N son alineados.

b) Los segmentos AB y CD son simétricos con respecto a la rectaMN.

c) La traslación que transformaD en C, transformaM en N.

d) La rotación de centro O y ángulo [AOD, transforma C en B.

e) La traslación de vector�!AC es igual a la traslación de vector

��!BD.

3. Considere el triángulo 4ABC isósceles en A, I el punto medio de BC, D = t�!AC

(B) y E =

t�!BI

(D). Estudie la naturaleza de los cuadriláteros ABDC yDECI.

4. Construya un triángulo 4ABC, en seguida el punto D, simétrico de A con respecto a B, y eltransformado E deB por la traslación de vector

�!AC. Mostrar que el triángulo4ABC es el trans-

formado del triángulo4BDE por una traslación la cual debe precisarse el vector.

5. Sea4ABC un triángulo rectángulo enB, yH la proyección ortogonal deB sobreAC. Se denotanpor A0 y C 0 los puntos tales que HBAA0 yHBCC 0 son dos paralelogramos. ¿Cuál es la naturalezadel cuadrilátero CAA0C 0?Sugerencia: U lice la traslación de vector

�!BH.

6. El cuadradoABCD es de centroO y de lado a. Determine los puntosM sobre el ladoAB,N sobreel lado CD, tales queMN k BC y que minimicen la poligonalOMNB, y exprese este mínimo enfunción de a.Pista: Sea O0 = T�!

BC(O).

7. Consdiere dos circunferencias de radios iguales � de centroO, y �0 de centroO0, que se cortan enA y en B. Considere dos rectas paralelas l y l 0:

l pasa por A, corta nuevamente a � en P , y a �0 en P 0;

l 0 pasa por B, corta nuevamente a � enQ, y a �0 enQ0.

Demuestre que PP 0Q0Q es un paralelogramo.

8. Demuestre que la composición de dos traslaciones es otra traslación.

4.11. Problemas de Simetrías Centrales o Reflexiones Puntuales

1. ¿Cuáles son las figuras simétricas de la figura siguiente con respecto a las cuatro esquinas delrectángulo?

51

Page 52: M Dosier M3 Trigonometria

2. Sean las rectasd yd 0 secantes enO y el punto I noperteneciente a ningunade las rectas anteriores.

a) Construya el punto intersección A de la recta d 0 con la imagen de la recta d por la simetríade centro I .

b) La recta AI corta a d en B . Precise la posición de los puntos I, A y B.

c) Aplicación: Construya un segmento de puntomedio I, con uno de sus extremos en d y el otroen d 0 .

3. Dado un triángulo4ABC, seaM el punto medio deBC.D y E son las proyecciones ortogonalesrespec vas de B y C sobre la mediana AM. U lizando propiedades de simetrías puntuales apli-cadas a la simetría respecto aM, demuestre que BD = CE, ie, los vér ces B y C equidistan dela mediana AM.

4. Sea ABCD un paralelogramo de centro O. Especificar las imágenes de los puntos A; B; C y Dpor S0.

5. El triángulo 4ABC es rectángulo en A; denotamos por D = SA (B) y E = SA (C). ¿Cuál es lanaturaleza del cuadrilátero BCDE?

6. Si ABCD y AECF son paralelogramos, ¿qué po de cuadrilátero es BEDF ?

7. Dado un paralelogramo ABCD de centro O, se trazan dos rectas paralelas l y l 0 pasando respec-vamente por A y C. Si l corta a la recta BC en E, y l 0 corta a la recta DA en F , determine la

naturaleza del cuadrilátero BEDF .

4.12. Problemas de Rotaciones

1. Dada la figura siguiente, construya las imágenes del cuadrado, triángulo y segmento obtenidos porla rotación de centro O y de ángulo 90� (en ambos sen dos).

O

2. Sean d una recta, O un punto no perteneciente a d y r la rotación de centro O y ángulo 120�, enel sen do directo. Construir, en el siguiente orden:

a) La proyección ortogonal de H deO sobre d .

b) La imagen H0 de H por r .

c) La imagen d 0 de d por r .52

Page 53: M Dosier M3 Trigonometria

d) Si denotamos por I el punto intersección de d y d 0, mostrar queO; I; H yH0 están sobre unmismo círculo.

3. Sean: � un círculo de centro O, r la rotación de centro O y ángulo de 50�. El punto A pertenece a�, denotemos por B la imagen de A por r y por C la imagen de B por r .

a) Ilustre la situación anterior.

b) Exprese en grados los ángulos del triángulo4ABC.

4. Trace el segmento AB de 4cm. Ubique el punto O de tal forma que la rotación de centro O yángulo 90�, muevan el punto A al punto B. ¿Cuál es la distancia del punto O a la recta AB.

5. a) ¿Cómo trazar los vér ces de un hexágono regular, u lizando sólo el compás?

b) Aplicación: Dados dos puntos O y A: Construir u lizando sólo el compás, el punto simétricode A con respecto aO y el transformado A por la rotación de centro O y ángulo 120�.

6. Sea el cuadrado ABCD de centro O en sen do directo. M un punto del segmento AD y N suimagen por una rotación de 90� en sen do directo y de centro O.

a) Demostrar que N pertenece al segmento AB.

b) Comparar las distancias deBM y CN, los ángulos\AOM y\BON y las áreas de los triángulos4BDM y4CAN.

7. Dada una recta l y un punto O que no pertenece a l , demostrar que las imágenes de l al hacertodas las rotaciones de centro O, son las tangentes a una circunferencia fija.

8. Dada una recta l y un punto O fuera de ella, se construye un triángulo equilátero 4OAB de talforma que A varía sobre l . ¿Qué lugar geométrico describe B?

9. Sobre los lados deunn-ágono regularA1A2 � � �An se dibujan puntosB1; B2; : : : ; Bn tal queA1B1 =

A2B2 = � � � = AnBn. Demuestre que B1B2 � � �Bn también es n-ágono regular.

10. Dado un cuadrado ABCD nombrado en sen do horario, sea I un punto en su interior tal que4DCI es equilátero, y J un punto en su exterior tal que4BCJ también es equilátero. Demuestreque A, I, J, están alineados. Para ello:

a) Construya las imágenes de J, I, A tras la rotaciónR�

3

C y demuestre que estas tres imágenesestán alineadas.

b) Concluya por propiedades de rotaciones que J, I, A, están alineados.

11. Los triángulos4ABC y4AB0C 0 son triángulos rectángulos isósceles y con la misma orientación.Demuestre que4ABB0 � 4ACC 0.

53

Page 54: M Dosier M3 Trigonometria

4.

Transformaciones en el plano: homoteclas

54

Page 55: M Dosier M3 Trigonometria

5.1. Teoremas Fundamentales

Las homotecias, dicho de forma sencilla, son las transformaciones geométricas básicas que es ran oencogen a las figuras geométricas, o dicho de manera un poco más formal, dilatan o contraen a las fi-guras geométricas; en casos muy excepcionales (y de mucho interés), la figura geométrica quedará delmismo tamaño. En cualquier caso, estas dilataciones o contracciones deben cumplir dos caracterís cas:Al transformar mediante una homotecia a una figura geométrica cualquiera, la figura resultante debeconservar las proporciones y la inclinación de la figura original.6

Definición 5.1. Homotecia: Dado un punto fijoO y un número real no nulo k , la homotecia de centroOy razón k , denotada por HO;k , es la transformación puntual que, a cada punto M asigna un punto M 0,tal que:

) Los puntos O,M yM 0 están alineados, y

) Los segmentos dirigidos OM yOM 0 sa sfacen la relación OM 0 = k �OM.

O equivalentemente, estas condiciones pueden abreviarse en la relación vectorial��!OM 0 = k

��!OM. El punto

O se llama Centro de Homotecia y el número real no nulo k se llama Razón de Homotecia.

Observaciones:

El número k puede tomar tanto valores posi vos como nega vos, esto se debe a que se estátrabajando con segmentos dirigidos, o con vectores si se prefiere.

Si k = 1, la homotecia HO;1 es la transformación iden dad, y todos los puntos son puntos fijos,es decir, HO;1(M) = M para todo M; en cambio, si k 6= 1, el único punto fijo es el centro dehomoteciaO.

Si k = �1, la homoteciaHO;�1 es la reflexión puntual respecto a O,RO.

Teorema 5.1. La homoteciaHO;k transforma al segmento dirigidoAB en un segmento dirigidoA0B0, talque:

1. A0B0 k AB, y

2. A0B0 = k � AB.6Esto es básicamente lo que un programa de computadora hace a una imagen cuando se u liza la función “zoom”.

55

Page 56: M Dosier M3 Trigonometria

O equivalentemente, transforma el vector�!AB en un vector

��!A0B0 tal que

��!A0B0 = k

�!AB.

Demostración. Se muestran dos soluciones, una por la vía sinté ca y otra u lizando vectores:

Primera demostración: Por la definción de la homotecia HO;k , los puntos O, A y A0 están alineados ylo mismo sucede con O, B y B0, entonces \AOB = \A0OB0; además, también por la definición dehomotecia se ene OA0

OA= k = OB0

OB, entonces4AOB ' 4A0OB0 (por el criterio lal de semejanza de

triángulos) con razón de semejanza k , y por lo tanto \OAB = \OA0B0 y A0B0

AB= k , que son proposicio-

nes equivalentes a A0B0 k AB y A0B0 = k � AB.

Segunda demostración: Por la definición de la homotecia HO;k se cumple que��!OA0 = k

�!OA y

��!OB0 =

k�!OB, entonces

��!A0B0 =

��!OB0 ���!OA0 = k

�!OB � k

�!OA = k

(�!OB ��!OA

)= k

�!AB.

Para completar las pruebas, si M es un punto cualquiera alineado con A y B, repi endo estas mismasdemostraciones,M 0 = HO;k(M) cumple queA0M 0 k AM yA0M 0 = k �AM, o equivalentemente

��!A0M 0 =

k��!AM, por lo queM 0 está alineado conA0 yB0; por lo tanto, el segmento dirigidoAB se transforma punto

a punto en el segmento dirigido A0B0, y lo mismo se dice con los vectores.

Corolario. La homotecia de una rectaAB es otra rectaA0B0 tal queAB k A0B0; en par cular, si el centrode homotecia pertenece a la recta AB, entonces la recta A0B0 coincide con la recta AB.

Demostración. Dejamos esta prueba como ejercicio.

Teorema 5.2. Recíproco: Dado un número real no nulo k , toda transformación que cumple que a cual-quier segmento dirigidoAB lo transforma en el segmento dirigidoA0B0 verificando las relacionesA0B0 kAB y A0B0 = k � AB, es una homotecia si k 6= 1, o es una traslación si k = 1.7

Demostración. Analizamos los casos por separado:

Caso 1: Si k 6= 1, definimos la homotecia HO;k como aquella que cumple A0 = HO;k(A); observe queHO;k(B) = B1 tal que A0B1 k AB y A0B1 = k � AB, por lo que B1 coincide con B0.

7En par cular, si k = 1 y A coincide con A0, entonces la transformación evidentemente es la transformación iden dad, lacual puede ser vista como una traslación de vector nulo o como una homotecia.

56

Page 57: M Dosier M3 Trigonometria

Caso2: Si k = 1, se ene que los segmentos dirigidos AB y A0B0 guardan las relaciones A0B0 k AB yA0B0 = AB, esta es justamente la definición de traslación. Es importante notar que este caso puede servisto comoel comportamiento al límite del caso anterior, si se inicia con un k 6= 1 y se define nuevamentela homoteciaHO;k como aquella que cumpleA0 = HO;k(A), y ahora se busca que k ! 1, será necesarioque O se mueva sobre la recta AA0 alejándose infinitamente de estos puntos, así las rectas AA0 y BB0

enden a ser paralelas y por tanto los segmentos AB y A0B0 enden a ser paralelos e iguales.

Corolario. Una homotecia está bien definida si se conocen las imágenes de dos puntos dis ntos dados.

Demostración. Dados los puntosA yB, siA0 = HO;k(A) yB0 = HO;k(B) son sus imágenes respec vas,entonces, por el teorema anterior, la razón de homotecia es k = A0B0

AB, y el centro de homotecia O es la

intersección de las rectas AA0 y BB0.8

5.2. Homotecia de figuras geométricas fundamentales

A con nuación se presentan como una serie de problemas guiados, los resultadosmás importantes de lahomotecia aplicada a rectas, triángulos y circunferencias, que son las figuras geométricas fundamentales.

PROBLEMA 5.1Homotecia de Rectas:La homotecia de rectas se divide en dos casos, que la recta pase por el centro de homotecia, o que larecta no pase por dicho centro.

1. Demuestre que si una recta pasa por el centro de una homotecia, la imagen de esta recta al apli-carle la homotecia es la misma recta.Observación: Esto no significa que los puntos no se han movido, sino que a pesar que se han mo-vido, tanto el punto preimagen como el punto imagen pertenecen a la misma recta.

2. Demuestre que si una recta no pasa por el centro de una homotecia, la imagen es otra recta para-lela a la original.

3. Ex enda estos resultados a la homotecia de planos.

4. Demuestre que la homotecia de un ángulo es otro ángulo con igual medida y orientación; si el vér-ce del ángulo coincide con el centro de homotecia y la razón de homotecia es posi va, entonces

el ángulo es invariante tras la homotecia.

5. Demuestre que la homotecia de un semiplano definido por una recta, es otro semiplano defini-do por la imagen de la recta dada; demuestre también que los semiplanos restantes también secorresponden.

PROBLEMA 5.2Homotecia de Triángulos:

8Note que por lo general, los segmentos dirigidos AB y A0B

0 son paralelos pero de dis nto tamaño, exceptuando el casocuando la homotecia es la transformación iden dad, pero este es un caso trivial en el que A0

B0 coincide con AB; así, por lo

general k 6= 1 y por tanto las rectas AA0 y BB0 son secantes.57

Page 58: M Dosier M3 Trigonometria

1. U lizando el problema anterior, demuestre que la homotecia de un triángulo es otro triángulo talque los lados correspondientes son respec vamente paralelos. Estos dos triángulos pueden sercoplanares, si el centro de homotecia se encuentra en el mismo plano que el triángulo original, ose ubican en planos paralelos en cualquier otro caso.

2. Demuestre por tanto que la homotecia transforma a un triángulo en otro triángulo semejante, yambos triángulos enen la misma orientación de los ángulos y la razón de homotecia k coincidecon la razónde semejanza. A este par de triángulos se les llama TriángulosHomoté cos. Demuestreademás que la razón entre las áreas de los triángulos homoté cos es k2.

3. Demuestre el recíproco: Dados dos triángulos semejantes con lados correspondientemente para-lelos (ie, triángulos homoté cos), existe una homotecia que transforma a un triángulo en el otro.Para ello, hay que demostrar que:

a) El centro de homotecia existe, es decir, que las tres rectas que unen vér ces correspondientes(aquellos que se oponen a lados correspondientemente paralelos) son concurrentes.

b) La razón de homotecia es la razón de semejanza de los triángulos.

4. Ex enda la homotecia a cuadriláteros, pentágonos, etc.

5. Demuestre que la homotecia del interior de un triángulo es el interior del triángulo imagen, yanálogamente, el exterior del triángulo se mapea al exterior del triángulo imagen. ¿Qué sucedecon el interior y el exterior de un polígono simple9?

PROBLEMA 5.3Homotecia de Circunferencias.

1. Demuestre que la homotecia de una circunferencia es otra circunferencia. Si el centro de homo-tecia está en el mismo plano que la circunferencia, la circunferencia resultante es coplanar a laoriginal; en cambio, si el centro de homotecia no está en el mismo plano que la circunferencia, lacircunferencia resultante está en un plano paralelo al plano de la circunferencia original.

2. Demuestre que si dos circunferencias dis ntas son concéntricas, el centro de éstas es un centrode homotecia que transforma una circunferencia en la otra.

3. Demuestre que si dos circunferencias no concéntricas enen radios iguales, el punto medio delsegmento que une los centros de las circunferencias es un centro de homotecia para éstas (dehecho, es una reflexión).

4. Dadas dos circunferencias no concéntricas y de radios dis ntos, siAB es un diámetro de la primeracircunferencia, A0B0 es un diámetro de la segunda circunferencia, y AB k A0B0, demuestre que

a) El punto de intersección de AA0 con BB0 es un centro de homotecia de las circunferencias.

b) El punto de intersección de AB0 con A0B es otro centro de homotecia de las circunferencias.

9Un polígono simple es aquel no ene auto-intersecciones.

58

Page 59: M Dosier M3 Trigonometria

5. Demuestre que en todos los casos, la razón de homotecia es igual a la razón de los radios.

6. Demuestre que el interior de una circunferencia se mapea al interior de la circunferencia imagen,y que el exterior se mapea al exterior.

7. Ex enda estos resultados a la homotecia de esferas.

PROBLEMA 5.4El Pantógrafo:

La siguiente figuramuestra un pantógrafo, un instrumento de diseño u lizado para aumentar o disminuirdibujos. El pantógrafo está compuesto por 4 varillas unidas por ar culaciones, y ene las siguientescaracterís cas:

El punto O está fijo, y el puntoM es variable.

Las varillas OA y AM enen igual longitud r . Esto le permite aM moverse 2r a la redonda de O.

El puntoB es un punto fijo sobre la varillaOA, pero escogido arbitrariamente. Este punto definirála escala.

El cuadrilátero ABM 0C es un paralelogramo de tal manera que O,M yM 0 están alineados.

Para u lizar el pantógrafo, debe colocarse un lápiz sobre el puntoM 0; al mover el puntoM describiendolos contornos de un dibujo, el puntoM 0 reproduce el dibujo a una escala 1 : OA

OB. Note que puede inter-

cambiarse los roles deM yM 0 para lograr un dibujo con una escala 1 : OBOA

.

También podemos construir un modelo de pantógrafo en la computadora, siguiendo los siguientes pa-sos:

1° En un programa de geometría, dibuje dos puntos cualesquiera, O yM.

59

Page 60: M Dosier M3 Trigonometria

2° Dibuje dos circunferencias de radio fijo r , una con centro en O y otra con centro enM.

3° El problema tendrá sen do únicamente si O yM están lo suficientemente cerca como para que lasdos circunferencias se corten, si esto no sucede, acerque un pocoM hacia O. Realizado esto, definaA como una de las intersecciones de las circunferencias.

4° Dibuje un puntoB sobre el segmentoOA. Este punto quedará fijo sobre el segmentoOA, esto quieredecir que siempre mantendrá constante las distancias hacia los extremos del segmento.

5° Por B, trace una recta paralela a AM y definaM 0 como la intersección de esta recta con OM.

6° El pantógrafo está listo, ac ve la opción de “Dejar rastro” a los punto M y M 0; a medida que Ud.muevaM, el programa le generará un dibujo a escala descrito porM 0.

Observe que para este modelo teórico de pantógrafo no es necesario construir el punto C, en el pantó-grafo real sí lo es porque no es posible ordenar a las varillas que se mantengan paralelas, entonces serecurre a un ar ficio mecánico; de cualquier forma, si Ud. desea dibunar el punto C, ene que trazaruna recta porM 0 paralela a OA, y la intersección de esta recta con AM define al punto C.

Finalmente, para consolidar todo lo anterior

1. Jus fique el funcionamiento del pantógrafo demostrando que para todo puntoM, se cumple queM 0 = HO;OB

OA

(M).

2. Elabore un pantógrafo en la computadora y verifique las propiedades de homotecia en el planopara rectas, triángulos y circunferencias; note que previo al diseño del pantógrafo, debe tener listala figura a la cuál se le aplicará la homotecia y obligar al puntoM a pertenece a dicha figura.

PROBLEMA 5.5Homotecia de una figura cualquiera:En general, dada una figura geométrica en el plano o en el espacio, es posible transformarla medianteuna homotecia y (según las discusiones previas) la figura resultante siempre será semejante a la original;por ejemplo, la homotecia de un pentágono es otro pentágono que, no necesariamente es del mismotamaño que el original, pero sí ene losmismos ángulos y sus lados son correspondientemente paralelosal original; esto mismo sucede con un arco de circunferencia, una fotogra a, un cilindro, uno cono, etc.Sin embargo, ¿cómo se puede llegar a una conclusión tan poderosa únicamente analizando la homoteciaaplicada a figuras tan elementales como la recta, los triángulos y las circunferencias?Para completar formalmente el argumento, es necesario recurrir al Cálculo; sin entrar en tantos detalles,en esencia, la idea es descomponer localmente cualquier curva en una línea poligonal de lados infini a-mene pequeños, una superficie en un poliedro triangular de caras infinitamente pequeñas, teselar unaregión plana con triángulos infinitamente pequeños, etc. y luego aplicar la homotecia a la poligonal (porejemplo) esperando que la poligonal imagen describa la curva imagen.Aunque esto es plausible, es importante establecerlo con más rigor; cualquiera que haya tenido expe-riencia con el Cálculo, sabrá que la idea que está de fondo es la con nuidad de la homotecia. Esta esuna propiedad, que dicho de manera sencilla establece que: dado un punto y un entorno cercano a ese

60

Page 61: M Dosier M3 Trigonometria

punto, la homotecia lo mapea en un entorno cercano a la imagen del punto. Para ello, aunque no esnada evidente, lo que debe probarse es dado un punto P y su imagen P 0 = HO;k(P ), si se contruyeuna circunferencia de radio arbitrario � y centro P 0, siempre es posible encontrar un radio � tal que laimagen tras la homotecia de la circunferencia de radio � y centro P está completamente contenida enla circunferencia de centro P 0 y radio �.Otra propiedad importante es que la homotecia es una transformación biyec va; esto resulta par cular-mente ú l cuando se quiere medir el ángulo entre la homotecia de dos curvas: Suponga que � y � sondos curvas que se cortan en un punto P , el ángulo entre � y � está definido como el ángulo que formanlas tangentes locales a las curvas en P ;10 suponga que �0, �0 y P 0 son sus respec vas imágenes tras unahomotecia.

En primer lugar observe que �0 y �0 se cortan en P 0.

En segundo lugar, � y �0 pueden verse como poligonales (llevadas al límite) que por propiedadesde homotecia, enen lados correspondientemente paralelos, entonces las curvas son localmenteparalelas.

En tercer lugar, si l es la recta tangente localmente a� en P , se transforma en una recta l 0 paralelaa l , que pasa por P 0, y si llegara a cortar (localmente) a l 0 en un segundo punto P 00, la preimagenforzosamente debería ser P , lo que contradice la inyec vidad de la homotecia; así, l 0 es tangentea �0 en P 0.

Finalmente, si m es la recta tangente a � en P , la imagen m0 es la recta tangente a �0 en P 0, porpropiedades de homotecia, el ángulo entre l ym es igual al ángulo entre l 0 ym0, entonces, el ánguloentre� y� es igual al ángulo entre�0 y�0. Por lo tanto, una figura, por complicada que sea, cumpleque su imagen tras una homotecia es semejante a la original, preservando la inclinación.

Para completar lo comentado en este párrafo, resuelva lo siguiente:

1. Demuestre que la homotecia es una transformación con nua.Se denota por C(P; r) al interior de una circunferencia (o una esfera) de centro P y radio r . Sea P 0

el imagen de un punto P tras la homoteciaHO;k . Demuestre que para un radio arbitrario � > 0, esposible construir convenientemente un radio � > 0 tal que HO;k (C(P; �)) está completamentecontenido en C(P 0; �).

2. Demuestre que la homotecia es una transformación biyec va.

EJEMPLO 5.1RECTA Y CÍRCULO DE EULER DE UN TRIÁNGULO. Demuestre que en todo triángulo, el centroide G estásituado entre el segmento OH, el cual une el circuncírculo y el ortocentro.La rectaOH es la recta de Euler del triángulo ABC y además

��!MO = �1

2

�!AH

10Observe que para definir la tangente local a una curva, es necesario hacer uso de límites, y la con nuidad de la hometeciaes nuevamente un tema central.

61

Page 62: M Dosier M3 Trigonometria

A

B C

P

M

NG

Sean M;N; P los puntos medios de los lados BC;CA y AB de un4ABC. El4MNP cuyos lados sonrespec vamente paralelos a cada uno de los lados del4ABC, es la transformación a través de una ho-motecia de razón NP

BC= �1

2.

El centro G de esta homotecia es común con el punto donde se cortan las rectas AM, BN y CP y estal que

��!GM = �1

2

�!GA, con esto se comprueba mediante homotecias la propiedad de la mediana de un

triángulo.Acabamos de establecer la homotecia HG;� 1

2, observemos que la altura trazada desde A del 4ABC ,

ene por homólogo a la altura trazada desdeM del4MNP , y esta se dirá que es la mediatriz de BC.El ortocentroH del4ABC ene por homólogo el centroO del cincuncírculo del4ABC y

�!GO = �1

2

�!GH

A

B

C

P

M

N

A0

H1

HO

G

Demostremos ahora que “En todo triángulo, los puntos medios de los lados, los pies de las alturas y lospuntos medios de los segmentos que unen al ortocentro con los vér ces del triángulo, están situados enunamisma circunferencia, que llamaremos circunferencia de Euler o circunferencia de los nueve puntos.PruebaSea w el centro de la circunferencia que pasa por los puntos M;N y P , este centro w es el homólogode O, que es el centro de la circunferencia que pasa por A;B y C y

��!Ow homólogo de

��!HO, de aqui que��!

Ow = �12

��!HO. Luego el punto w es el punto medio deOH. La igualdad

��!Hw = 1

2

��!HO muestra queH es

el centro de la homotecia de las circunferencias ABC yMNP , cuya razón es posi va.Veamos ahora que con la homotecia encontradaHH; 1

2la transformación del punto A es el punto medio

� de AH, y la transformación del puntoH1 de la recta AH corta a la circunferencia ABC en el pie A0 de

62

Page 63: M Dosier M3 Trigonometria

la altura AH.Los puntos � y A0 pertenecen a la circunferenciaMNP .

A

B

C

P

M

N

A0

H1

C 0

B0HO

G

J

!

5.3. Problemas de Homotecias

1. Dos circunferencias y 0 son tangentes en I. Se considera las cuerdasMN en la circunferencia de longitud fija. Las rectas MI y NI cortan nuavamente a 0 en M 0 y N 0, respec vamente.Demuestre que la rectaM 0N 0 permanece tangente a una circunferencia fija.

2. Sean � y �0 dos circunferencias tangentes en T , y seaA un punto interior a �. Dado un punto varia-bleM sobre �, se defineN como la otra intersección deAM con �, luegoM 0 es la otra intersecciónde MT con �0, y N 0 es la otra intersección de NT con �0. Demuestre que las rectas M 0N 0 pasanpor un punto fijo.

3. Una circunferencia variable es tangente a una recta l en unpunto fijoO de l . ¿Cuáles son los lugaresgeométricos de los puntos de contacto en el círculo generados por rectas tangentes paralelas a unarecta fijam?

4. Dados dos cuadrados de lados correspondientemente paralelos, ¿qué sucede con las rectas queunen vér ces correspondientes? ¿podemos decir lo mismo de dos rectángulos de lados corres-pondientemente paralelos?

5. ABCD y A0B0C 0D0 son dos rectángulos tales que los lados más largos de cada uno, AB y A0B0, seubican sobre una misma recta l . Demuestre que los rectángulos son semejantes (el cociente entreel largo y el ancho es igual) si y sólo si las rectas BB0, CC 0 y l concurren.

6. Dadas dos triángulos rectángulos isósceles no necesariamente iguales, demuestre que si sus hipo-tenusas se ubican sobre una misma recta, existe una homotecia que transforma a un triángulo enel otro. Dé una manera de construir el centro de homotecia.

63

Page 64: M Dosier M3 Trigonometria

7. El problema de los bordes: Dado un polígono, se construye otro polígono en su interior obtenidoal bordear los lados con una banda de ancho constante a. El problema estudia lo siguiente: ¿Lasrectas que unen vér ces correspondientes AA0, BB0, CC 0; : : : son concurrentes?

En la figura anterior se muestra el caso cuando el polígono original es un pentágono. Demuestreque

a) Si el polígono original es un triángulo, la respuesta es SÍ.

b) Si el polígono original es un cuadrado, la respuesta también es SÍ.

c) Si el polígono original es un rectángulo pero no es cuadrado, la respuesta es por lo gene-ral NO. Dentro de este caso, ¿cuándo la respuesta será SÍ? Generalice sus ideas para otroscuadriláteros.

d) Si el polígono original es un rombo, la respuesta es sí. En general, demuestre que si el polí-gono original es convexo y admite una circunferencia tangente a todos sus lados, entoncesla respuesta será SÍ.

e) Si existe una homotecia que transforma a unpolígono en el otro la respuesta es SÍ, y viceversa.

f ) ¿Qué sucede cuando el polígono original es regular?

8. Dado un triángulo4ABC y un puntoM sobre el ladoBC, seanM1 la proyección ortogonal deMsobre AB,M2 la proyección ortogonal deM1 sobre CA, y P 0 la proyección ortogonal deM2 sobreBC. El obje vo del problema es determinar todos los puntosM (si existen) tales que el recorridoregresa aM, ie,M 0 = M. Para ello:

1° Tome un punto cualquiera P sobre BC y construya P1, P2 y P 0.

2° Defina I como la intersección de las rectas PP1 y P2P0, y J como la intersección de las rectas

AI con BC.

3° Verifique que J es una solución al problema (aplique una homotecia con centro en A y quetransforme a I en J).

64

Page 65: M Dosier M3 Trigonometria

4° En el sen do lógico contrario: Suponga queM es un punto solución del problema, demuestreque a par r del recorrido que determinaM es posible reconstruir el recorrido que determinacualquier otro punto P (aplique una homotecia convenientemente).

9. Demuestre que toda homotecia con razón nega va puede ser escrita como la composición de unahomotecia con razón posi va y una reflexión.

10. Demuestre queHB; 1k

� HA;k es una traslación. ¿Qué traslación es?

11. Demuestre que toda homotecia HO;k es equivalente al producto de la homotecia HO1;k por unatraslación paralela a

��!OO1. Para ello, siga los siguientes pasos:

a) Dado un punto cualquiera A, construya A0 = HO;k(A).

b) Construya A00 = HO1;k(A) yO00 = HO1;k(O).

c) Demuestre que OA0A00O00 es un paralelogramo y concluya.

12. Dada una homotecia no trivial (razón de homotecia k 6= 1), demuestre que el centro de homoteciaes el único punto fijo y que las rectas que pasan por dicho centro son las únicas rectas fijas tras lahomotecia.11

13. Demuestre que la homotecia es una transformación directa, es decir, no cambia la orientación delos ángulos.

14. Demuestre que una homotecia con razón de homotecia k = 1 es la transformación iden dad.

15. Demuestre que la transformación inversa de la homotecia HO;k es otra homotecia, la cual estádefinida porHO; 1

k

.

16. Demuestre que el conjunto de homotecias y traslaciones cumplen tener estructura de Grupo conla composición de funciones como operación, es decir, forman un Grupo de Transformaciones.

17. Dado un punto arbitrarioO, demuestre que el conjunto de homotecias concéntricas enO formanun Grupo de Transformaciones.12

18. Demuestre que el producto de tres homotecias es o bien una homotecia o una traslación. Gene-ralice el resultado para 4; 5; : : : homotecias.

19. Sea � la transformación iden dad; se dice que una transformación � es involutoria si �2 = �.Determine todas las homotecias que son involutorias.

20. Demuestre que, dada una homoteciaH, el entero posi vo n más pequeño para el cualHn = �

a) es 1 si y sólo siH = �,

b) es 2 si y sólo si la razón de homotecia es�1 (reflexión puntual), y

c) no existe en cualquier otro caso.

11Punto o recta fija tras la homotecia quiere decir que al aplicar la homotecia a dicha figura, el resultado (o imagen) es lamisma figura.

12Dos homotecias son concéntricas si comparten el centro de homotecia.65

Page 66: M Dosier M3 Trigonometria

21. Demuestre que la homotecia de una recta o un plano que pase por el centro de homotecia, es lamisma figura.

22. Demuestre que la homotecia de una circunferencia o una esfera con respecto a su centro, es unacircunferencia o una esfera concéntrica.

23. U lizando las propiedades de las homotecias, demuestre el Teorema de Menelao:

a) Suponga que el4ABC es cortado por la recta transversalO1O2O3, tal queO1 está sobreAB,O2 sobreBC yO3 sobreCA. Calcule el valor de la razón de homotecia k1 tal queHO1;k1(A) =

B; calcule también el valor de la razón de homotecia k2 para queHO2;k2(B) = C.

b) Demuestre queHO2;k2 �HO1;k1 es otra homotecia que transformaA enC; determine ademáscuál es centro y la razón de esta nueva homotecia.

c) Demuestre el Teorema de Menelao

AO1

O1B

BO2

O2C

CO3

O3A= �1

24. Demuestre el Teorema de Varignon u lizando propiedades de homotecias: Dado un cuadriláterocualquiera ABCD, sean P Q, R, S, los puntos medios de AB, BC, CD,DA, respec vamente.

a) Construya una homotecia que transforme al segmento PQ en el segmento AC.

b) Construya una homotecia que transforme al segmento AC en el segmento SR.

c) Averigüe qué po de transformación es la composición de estas dos homotecias.

d) A par r de lo anterior, concluya que ABCD es un paralelogramo.

e) DefiniendoM yN como los puntos medios de AC yBD, respec vamente, repita el procesoanterior para demostrar que PMRN yQNSM son paralelogramos también.

f ) Demuestre además que PR, QS y MN concurren. Este punto es llamado el Centroide deABCD.

25. Dado un triángulo4ABC seaM un punto variable sobre el ladoAB.N es la intersección de laACcon la recta que pasa porM y es paralela a BC. P es la intersección de BN con CM. Demuestreque B describe una mediana del triángulo ABC cuandoM varía.

26. Sean ABCD y AB0C 0D0 dos paralelogramos con igual orientación. Demuestre que BD k B0D0.

27. SeaABCD un cuadrilátero concíclico convexo. Demuestre que las cuatro rectas trazadas con estassiguientes caracterísicas son concurrentes:

pasa por el punto medio de un lado del cuadrilátero, y

es perpendicular al lado opuesto.

Pista: LlamandoK a la intersección de las cuatro rectas del problema, busque la relación que existeentre el circuncentro, el centroide yK.

66

Page 67: M Dosier M3 Trigonometria

28. Los vér ces B y C de un triángulo 4ABC están fijos, determine el lugar geométrico que des-criben el centroide y los puntos medios de los lados AB y AC si A varía sobre una recta o unacircunferencia.

29. En un cuadrilátero ABCD, los vér ces A, B y C están fijos, y el puntoD se mueve de tal maneraque el lado AD siempre ene la misma medida. Determine el lugar geométrico de

a) los puntos medios de AD, BD y CD,

b) el punto medio del segmentoRS, que ene por extremos a los puntos medios de AC yBD,respec vamente.

30. Resuelva el problema anterior, pero suponiendo que D se mueve de tal forma que el \ADB seman ene constante.

31. Sean l ym dos rectas fijas que se cortan en O; desde un punto P se construyen las proyeccionesA yB a l ym, respec vamente. ¿Qué lugar geométrico describe P si la recta AB varía man endouna inclinación constante.

32. Encuentre una homotecia dis nta de la iden dad, que transforme a un paralelogramo en símismo.

33. Encuentre dos homotecias dis ntas de �, que dejen invariantes a las bases de un trapecio.

34. Circunferencias tangentes, rectas tangentes y centros de homotecia:

a) Dada una circunferencia �, si P es un punto cualquiera sobre � y k 6= 1, demuestre queHP;k(�) es una circunferencia tangente a � en P (si k > 0, las circunferencias son tangentesinternas, mientras que si k < 0 son circunferencias tangentes externas). ¿Cuál es el radio dela circunferencia resultante?

b) Demuestre el recíproco del resultado anterior: si dos circunferencias son tangentes en P (yasea circunferencias tangentes internas o externas), existe una homotecia de centro P y razónk 6= 1 que transforma a una circunferencia en la otra. ¿Cuál es la razón de homotecia?

c) Demuestre que si las circunferencias son tangentes externas en P , el punto de intersecciónde las dos rectas tangentes comunes que no pasan por P es el otro centro de homotecia delas circunferencias. ¿Cuál es la razón de homotecia?

d) Dadas dos circunferencias una fuera de la otra, demuestre que las dos rectas tangentes co-munes externas concurren en un centro de homotecia de las circunferencias (a menos quelos radios sean iguales, en tal caso, estas tangentes quedan paralelas), y que las dos rectastangentes comunes internas concurren en el otro centro de homotecia de las circunferencias.

e) Dadas dos circunferencias de radios dis ntos y secantes en dos puntos dis ntos, demuestreque las dos rectas tangentes comunes se cortan en un centro de homotecia de las circunfe-rencias.

35. Denote por d(P; l) la distancia de un punto P a una recta l . Sean l y m dos rectas fijas y k unaconstante.

a) Si l ym son rectas secantes en un puntoO, demuestre que el lugar geométrico de los puntosP tales que d(P;l)

d(P;m)= k , son dos rectas que pasan por O.

67

Page 68: M Dosier M3 Trigonometria

b) Si l k m, el lugar geométrico descrito anteriormente, es una recta paralela a l ym, y se ubicade tal forma que el cociente de las distancia a l ym es igual a la constante k .

c) Si en par cular k = 1, las rectas resultantes son la bisectriz interna y la bisectriz externa delángulo formado por l ym.

d) ¿Cuál es el lugar geométrico de los centros de las circunferencias tangentes simultáneamentea l ym?

36. Centros de homotecia de tres circunferencias:

a) Dadas dos circunferencias no concéntricas, demuestre que sus centros de homotecia dividenarmónicamente al segmento que une los centros de las circunferencias.

b) Dadas tres circunferencias cuyos centros forman un triángulo, demuestre que los seis cen-tros de homotecia de tomar dos a dos a las circunferencias, forman cuatro ternas de puntosalineados.

37. U lizando propiedades de triángulos homoté cos, demuestre que las tres medianas de un trián-gulo concurren.

38. Recta de Euler: Demuestre u lizando homotecia que el ortocentro, el circuncentro y el centroidede un mismo triángulo, están alineados.Pista: note que el triángulo medial es homoté co al triángulo original.

39. Circunferencia de los 9 puntos: Dado un4ABC, sea � el circuncírculo yH el ortocentro. Demues-tre queHH; 1

2(�) es la circunferencia de los 9 puntos del4ABC. ¿Qué propiedad ene el centro

de la circunferencia de los 9 puntos? ¿Cuáles son los dos centros de homotecia de estas dos cir-cunferencias?Pista: Demuestre que la reflexiones del ortocentro con respecto al los lados y con respecto a lospuntos medios de los lados, pertenecen a �.

40. Una circunferencia variable es tangente a una recta fija l en un punto fijo A. Determinar el lugargeométrico de los puntos de contacto de las rectas tangentes a la circunferencia con inclinaciónfija.

41. Una circunferencia variable es tangente a dos rectas secantes fijas. Determinar el lugar geométricode los puntos de contacto de las rectas tangentes a la circunferencia con inclinación fija.

42. Dos circunferencias fijas de radios dis ntos, son tangentes enA. SeaAB una cuerda variable sobreuna circunferencia, y AC una cuerda variable sobre la otra cirunferencia, tal que AB ? AC.

a) Demuestre que la recta variable BC pasa por un punto fijo.

b) Determine el lugar geométrico que describe el punto medio de BC.

c) Determine el lugar geométrico que describe la proyección de A sobre BC.

43. Construya una circunferenia cumpliendo los requisitos de cada uno de los siguientes casos:

68

Page 69: M Dosier M3 Trigonometria

a) Dada una circunferencia �, una recta l y un puntoA que puede estar ubicado sobre � o sobrel , construya una circunferencia tangente a � y tangente a l , de tal manera queA sea un puntode tangencia.

b) Construya una circunferencia que pase por un punto dado y que sea tangente a dos rectassecantes dadas.

c) Construya una circunferencia que sea tangente a una circunferencia dada y a dos rectas se-cantes dadas.

44. Construya el triángulo4ABC en cada uno de los casos siguientes:

a) Conociendo el ángulo en A y las medianas respec vas a B y C.

b) Conociendo los ángulos B y C, y la distancia OH.

c) Conociendo el vér ce A, el centroide G y sabiendo que B y C pertenecen a dos rectas (ocircunferencias) dadas.

d) Conociendo la razón de los lados AB y AC, el ángulo A y el perímetro del triángulo.

e) Conociendo los lados AB y AC y la bisectriz interna AD.

45. Un Cuadrilátero Ortocéntrico es la figura formada por los vér ces de un triángulo y su ortocentro.

a) Demuestre que en un cuadrilátero ortocéntrico, cada vér ce es el ortocentro del triánguloformado por los tres vér ces restantes.

b) Demuestre que los cuatro centroides de los triángulos formados por los vér ces de un cua-drilátero ortocéntrico, forman otro cuadrilátero ortocéntrico homoté co al original.

c) Demuestre que la circunferencia de los 9 puntos de un cuadrilátero ortocéntrico es concén-trica a la circunferencia de los 9 puntos del cuadrilátero formado por los cuatro centroides.

5.4. Composición de Homotecias

Teorema 5.3. Producto de dos Homotecias: Dadas dos homoteciasHO1;k1 yHO2;k2 , si alguna de ellas esla transformación iden dad, el producto de éstas es de nuevo una homotecia y dicho producto es ademásconmuta vo; en cualquier otro caso, se cumple que:

1. Independientemente de los valores que tome k1 o k2, si O1 coincide con O2, el productoHO2;k2 �HO1;k1 es otra homotecia HO;k1k2 cuya razón de homotecia es k1k2 y su centro de homotecia esO = O1 = O2. En este caso, las homotecias son llamadas homotecias concéntricas, y el productoes conmuta vo, es indiferente cuál de las dos homotecias se realiza primero.

2. Si k1 6= 1, k2 6= 1 y O1 no coincide con O2, el producto no es conmuta vo, y se subdivide en doscasos:

a) Si k1k2 6= 1, el productoHO2;k2 � HO1;k1 es otra homoteciaHO;k1k2 cuya razón de homoteciaes k1k2, su centro de homoteciaO se encuentra sobre la rectaO1O2 y cumple O1O

OO2= k2�1

k2(k1�1) .

b) Si k1k2 = 1, el producto HO2;k2 � HO1;k1 es una traslación cuyo vector es (k1�1k1

)���!O1O2, o

equivalentemente (1� k2)���!O1O2.

69

Page 70: M Dosier M3 Trigonometria

Demostración. Si alguna de las homotecias es la transformación iden dad, el resultado es evidente. Porotra parte, el siguiente argumento es válido para el resto de casos, por comodidad se u lizará notaciónvectorial, pero la prueba sinté ca es muy parecida y se deja como ejercicio:

Dado el vector�!AB, HO1;k1(

�!AB) =

���!A1B1 con

���!A1B1 = k1

�!AB, y HO2;k2(

���!A1B1) =

���!A2B2 con

���!A2B2 =

k2���!A1B1, entonces

���!A2B2 = k1k2

�!AB. Esto implica queHO2;k2 � HO1;k1 es una homotecia de razón k1k2 a

menos que k1k2 = 1, en cuyo caso es una traslación.

A con nuación se completan las pruebas

1. Sin importar los valores que tome k1 o k2, si O = O1 = O2, la homoteciaHO;k1k2 ene el mismoefecto neto queHO2;k2 � HO1;k1 y queHO1;k1 � HO2;k2 , esto completa la prueba.

2. En cambio, k1 6= 1, k2 6= 1 yO1 no coincide con O2

a) Si k1k2 6= 1, ya se demostró que el producto de las dos homotecias dadas es otra homotecia;a con nuación se demostrará queO, el centro de esta nueva homotecia, pertenece a la rectaO1O2 y cumple la relación O1O

OO2= k2�1

k2(k1�1) ; de nuevo, se hace tanto la prueba sinté ca comola vectorial:

Primera demostración: Se supone conocida la ubicación deO, el centro de la homotecia pro-ducto. Sea O0 = HO1;k1(O), dado que O es punto fijo en la homotecia producto, debe cum-

plirse que O = HO2;k2(O0). Por la definición de estas homotecias,

���!O1O

0 = k1��!O1O y

��!O2O =

k2���!O2O

0 = k2

(��!O2O +

��!OO1 +

���!O1O

0), entonces

��!O2O = k2

��!O2O + k2

��!OO1 + k2k1

��!O1O, de

donde (1� k2)��!O2O = k2(k1 � 1)

��!O1O, por lo tanto

��!O1O��!OO2

=k2 � 1

k2(k1 � 1)

lo cual demuestra queO1,O2 yO están alineados (los vectores son múl plos escalares) y larelación buscada.

70

Page 71: M Dosier M3 Trigonometria

Segunda demostración: La demostración anterior puede realizarse también sinté camente,sin embargo, se mostrará una demostración alterna va. Dado un punto A que no pertencea la recta O1O2, sea A1 = HO1;k1(A) y A2 = HO2;k2(A1); la homotecia producto, que trans-forma A en A2, cumple que su centro de homotecia está sobre la recta AA2, llamemosOA ala intersección de AA2 con O1O2, por el teorema de Menelao aplicado al4O1A1O2 con latransversal AA2OA se ene(

O1A

AA1

)(A1A2

A2O2

)(O2OA

OAO1

)= �1

pero O1A1

O1A= k1 lo cual implica O1A

AA1= 1

k1�1 ; análogamente, como O2A2

O2A1= k2 entonces A1A2

A2O2=

1�k2k2

, y sus tuyendo (1

k1 � 1

)(1� k2

k2

)O2OA

OAO1

= �1O2OA

OAO1

= �(k1 � 1)k21� k2

O1OA

OAO2

=k2 � 1

k2(k1 � 1)

Si ahora se toma un segundo puntoB que no pertenezca aO1O2 y se repite el mismo proce-dimiento, definendoB2 = HO2;k2 �HO1;k1(B) yOB como la intersección deBB2 conO1O2,se puede demostrar que

O1OB

OBO2

=k2 � 1

k2(k1 � 1)

por lo que OA = OB, y dado que el centro O de la homotecia producto se encuentra tantoen la recta AA2 como en la recta BB2, debe cumplirse que O = OA = OB y por tanto

O1O

OO2

=k2 � 1

k2(k1 � 1)

La relación buscada.

Ahora se discu rá si el producto de dos homotecias no concéntricas es conmuta vo o no loes en este caso. Note que si se invierte el orden del producto, los roles de k1 y k2 se invierten

71

Page 72: M Dosier M3 Trigonometria

y por tanto la homotecia resultante en este caso tendría como centro a un punto O� tal queO1O

O�O2= k1�1

k1(k2�1) ; para que la homotecia producto sea comnmuta va, O� debería coincidircon O, pero eso equivale a

k2 � 1

k2(k1 � 1)=

k1 � 1

k1(k2 � 1)

Desarrollando esta expresión y simplificando, se llega a la condición necesaria k1k2 = 1, locual de por sí contradice la hipótesis, sin embargo, si todavía insi éramos, al sus tuir estacondición en la relación que probamos13

O1O

OO2

=k2 � 1

k2(k1 � 1)=

k2 � 1

k2k1 � k2=

k2 � 1

1� k2= �1

Pero esto obliga a que O se aleje infinitamente de O1 y O2 y dicho producto se deforma enuna traslación, tal comos se dijo en un teorema anterior. En conclusión, si los dos centros dehomotecias no coinciden, k1 6= 1, k2 6= 1 y k1k2 6= 1, el producto de estas homotecia no esconmuta vo.

b) Si k1k2 = 1, ya se demostró que el producto de las dos homotecia es una traslación, resulta-do que se confirmó nuevamente en el el párrafo anterior. A con nuación se demostrará quela traslación ene por vector a (k1�1

k1)���!O1O2.

Primera demostración: Dado un punto A, sean A1 = HO1;k1(A) y A2 = HO2;k2(A1); si Apertence a O1O2, tanto A1 como A2 también pertenecen a O1O2 y el resultado se ob enerapidamente haciendo un poco de álgebra; por otra parte, si A no pertence a O1O2, por ladefinición de homotecia, O1A1 = k1 �O1A yO2A2 = k2 �O2A1, pero dado que k1k2 = 1 seene

O1A1

O1A= k1 =

1

k2=

O2A1

O2A2

U lizando el recíproco del teorema de Thales aplicado al4O1A1O2 se concluye queO1O2 kAA2; observe4O1A1O2 ' 4AA1A2, y la razón de semejanza es

O1O2

AA2

=O1A1

AA1

=k1

k1 � 1

Esto significa que independientemente del puntoA que se escoja,AA2 es un segmento para-lelo aO1O2 y su magnitud sólo depende de k1 yO1O2, es decir, es constante; esto se resumeen la relación vectorial

��!AA2 =

k1�1k1

���!O1O2; por lo tanto, el producto de las homotecias es una

traslación que ene al vector k1�1k1

���!O1O2 como parámetro.

13Observe que k2 6= 1 porque en caso contrario k1 = 1 dado que k1k2 = 1 y ambas homotecias serían la transformacióniden dad.

72

Page 73: M Dosier M3 Trigonometria

73

Page 74: M Dosier M3 Trigonometria

Segunda demostración: De nuevo, se toma un punto A y las imágenes A1 y A2, tal comoantes, sabiendo que

���!O1A1 = k1

��!O1A,

���!O2A2 = k2

���!O2A1 y k2 = 1

k1.

��!AA2 =

��!AA1 +

���!A1A2

=(���!O1A1 �

��!O1A

)+

(���!A1O2 +

���!O2A2

)=

(���!O1A1 � 1

k1

���!O1A1

)+(���!A1O2 + k2

���!O2A1

)=

(1� 1

k1

)���!O1A1 +

(1� 1

k1

)���!A1O2

=

(1� 1

k1

)(���!O1A1 +

���!A1O2

)=

(k1 � 1

k1

)���!O1O2

La relación buscada.

Finalmente, si se invierte el orden del producto de las homotecias, haciendo un razonamientoanálogo se llegaría a una traslación con vector k2�1

k2

���!O2O1, para que el producto sea conmu-

ta vo, debería cumplirse que esta úl ma traslación coincide con la anterior, pero eso sólo sedará si

k1 � 1

k1=

1� k2

k2

Lo cual, dado que k1k2 = 1 es equivalente a (k1 � 1)2 = 0, es decir, k1 = 1, obligandotambién a k2 = 1, por lo que se cae en el caso trivial cuando las dos homotecias son trans-formaciones iden dad. En conclusión, si O1 no coincide con O2, k1 6= 1, k2 6= 1 y k1k2 = 1,el producto tampoco es conmuta vo.

Teorema 5.4. El producto en cualquier orden de una homotecia con una traslación, es otra homoteciacon la misma razón.

Demostración. SeanHO;k y T�!� la homotecia y la traslación que se u lizarán en la composición; sea�!AB

un vector cualquiera,

HO;k � T�(�!AB

)= HO;k

(T�

(�!AB

))= HO;k

(�!AB

)= k

�!AB

74

Page 75: M Dosier M3 Trigonometria

Dadoque la traslación deja invariante al vector�!AB y la homotecia losmul plica por un escalar; de aquí se

concluye que la transformación producto debe ser una homotecia con razón k , y el centro de homoteciase encuentra en la intersección de las rectasAA0 yBB0, dondeA0 = HO;k �T� (A) yB0 = HO;k �T� (B).

Si se intercambia el orden y se realiza la composición T� � HO;k , análogamente se demuestra que estatransformación debe ser otra homotecia de razón k y se deja como ejercicio.

75